NOTEPAD
Results
of 200 questions answered correctly

You have reached of 200 points, ( %)

Your time

Question 1 of 200

1. A patient had to go through an operation. Doctors introduced him dithylinum (listenone) and performed intubation. After the end of operation and cessation of anesthesia the independent respiration wasn’t restored. Which enzyme deficit prolongs the action of muscle relaxant?

Explanation

Dithyllium is a depolarizing myorelaxant (neuromuscular blocking drug). They are used as adjuvant drugs in anaesthesia during surgery to relax skeletal muscles. An absence or deficiency of acetylcholinesterase/pseudocholinesterase enzyme leads to a silent condition that manifests itself only when people that have the deficiency receive the muscle relaxants. This enzyme plays an important role in  the metabolism of ester-based local anesthesics, a deficiency of blood cholinesterase can prolong the effect of dithylinium from just 5-7minutes to 8hours.

2.

A patient is ill with diabetes mellitus that is accompanied by hyperglycemia of over 7,2 millimole/l on an empty stomach. The level of what blood plasma protein allows to estimate the glycemia rate retrospectively (4-8 weeks before examination)?

Explanation

 

3. A 60 y.o. patient has a reduced perception of high-frequency sounds. What structures’ disorder of auditory analyzer caused these changes?

Explanation

Two membranes divide the spiral canal of cochlea into 3 compartments – vestibular and basilar membrane. Along the basilar membrane are 20,000-30,000 tiny fibers called basilar fibers. Each fiber has different size and shape. Fibers near the oval window (base) are short and stiff (narrowest), while approaching towards helicotrema (apex), the basilar fibers gradually become longer and soft (widest). The motion of the basilar membrane is generally described as a travelling wave. The parameters of the membrane (stiffness, narrow, wide) at a given point along its length determine its characteristic frequency at which it is most sensitive to sound vibrations. High frequency sounds localize near the base of the cochlea (near the round and oval windows), while low frequency sounds localize near the apex (helicotrema); and the middle frequency sounds – middle part of helix.

There is an impaired perception of high frequency sounds; that means there is damage near the base of cochlea (i.e. near the round and oval windows).

4.

A patient ill with diabetes mellitus felt acute pain in his right foot. Objectively: foot thumb is black, foot tissues are edematous, there are foci of epidermis desquamation, stinking discharges. What clinicopathological form of necrosis is it?

Explanation

       Gangrene is a morphological type of necrosis. It develops in organs and tissues having contact with environment. The most often examples of gangrene are gangrene of the lower extremities, uterus, lungs etc. There are 3 main forms of gangrene namely: dry, wet (moist) and gas gangrene. Wet gangrene macroscopically, is moist soft swollen (edematous) rotten and black. It is usually applied to a limb, generally the lower leg, that has lost its blood supply and has undergone necrosis (typically coagulative necrosis) involving multiple tissue planes. Gangrene of lower extremities, as a result of advanced vascular disease is about 100 times more common in diabetes than in the general population.

5. A patient with a stab wound of the anterior stomach wall is in surgical care. What formation of abdominal cavity did the stomach contents get into?

Explanation

        In the superior level of the peritoneal cavity, 3 bursae are found: the hepatic, pregastric (antegastric) and the omental bursa are distinguishable (first two refer to clinical anatomy rather than to normal anatomy). Walls of the pregastric (antegastric) bursa are:

·        Medial – falciform ligament

·        Posterior – anterior wall of stomach, lesser omentum, left portion of the coronary ligament and the left triangular ligament.

·        Superior, lateral and anterior – parietal peritoneum

·        Inferior – transverse mesocolon.

Since this bursa is found anterior to the stomach. It therefore means that a stab wound of the anterior stomach wall will make the contents of the stomach to get into the pregastric (antegastric) bursa.

Omental bursa bursa lies posterior to the stomach. Hepatic bursa embraces the right lobe of liver, gallbladder, upper pole of right kidney and right suprarenal gland.

6.

A 4 y.o. child with signs of durative proteinic starvation was admitted to the hospital. The signs were as follows: growth inhibition, anemia, edemata, mental deficiency. Choose a cause of edemata development:

Explanation

The liver is the main site of protein synthesis i.e. synthesis of albumin, globulin, fibrinogen etc., which are necessary to maintain plasma oncotic pressure. In a situation of protein starvation, there is decrease of protein synthesis which results in decreased oncotic pressure and then, edema. Therefore, an impairment of protein synthetic function of liver leads to the formation of edema.

7. Autopsy of a patient who suffered from croupous pneumonia and died from pneumococcal sepsis revealed 900 ml of turbid greenish-yellow liquid in the right pleural cavity. Pleural leaves are dull, plephoric. Name the clinicopathological form of inflammation in the pleural cavity:

Explanation

       Complications of croupous or lobar pneumonia includes empyema, gangrene, carnification, abscess formation, bacteremic spread which leads to purulent meningitis, bacterial endocarditis, arthritis, pericarditis and other organs. Empyema is a purulent inflammation of serous membranes (empyema of pleura, empyema of gall bladder and urinary bladder etc). Purulent or suppurative inflammation is characterized by the production of large amounts of pus or purulent exudates (liquids) consisting of a lot of neutrophils, necrotic cells and edema fluid.

8.

Examination of a patient revealed reduced contents of magnesium ions that are necessary for attachment of ribosomes to the granular endoplasmatic reticulum. It is known that it causes disturbance of protein biosynthesis. What stage of protein biosynthesis will be disturbed?

Explanation

A large number of components are required for translation (synthesis of a protein). These include all the amino acids that are found in the finished product, the mRNA to be translated, tRNA, functional ribosomes, energy sources and enzymes as well as protein factors needed for initiation, elongation and termination of the polypeptide chain. In eukaryotic cells, the ribosomes are either “free” in the cytosol or are in close association with the endoplasmic reticulum (which is then known as the “rough” endoplasmic reticulum or RER). The RER – associated ribosomes are responsible for synthesizing proteins that are to be exported from the cell as well as those that are destined to become integrated into plasma, endoplasmic reticulum or golgi membranes or incorporated into lysosomes.

9.

Analysis of blood serum of a patient revealed increase of alanine aminotransferase and aspartate aminotransferase level. What cytological changes can cause such a situation?

Explanation

Alanine aminotransferase is present in the cytosol, while aspartate aminotransferase is present in the mitochondria. Increased serum levels of alanine aminotransferase (ALT) and Aspartate aminotransferase (AST) can indicate cell necrosis (cellular breakdown). They can be used as a diagnostic tool.

10. Power inputs of a boy increased from 500 to 2000 kJ pro hour. What can be the cause of it?

Explanation

The best answer is physical exercise considering the enormous energy level increase in an hour. All other options can cause energy level increase but physical exercise certainly requires a higher level of energy input.
11. A patient presents high activity of LDH1,2, aspartate aminotransferase, creatine phosphokinase. In what organ (organs) is the development of a pathological process the most probable?

Explanation

In laboratory diagnosis of acute myocardial infarction: Creatine kinase isoenzyme MB (CK-MB) appears within 4-8hrs, peaks at 24hrs and disappears within 1.5-3 days. Sensitivity and specificity is 95%. Reappearance of CK-MB after 3days – reinfarction. CK-MM – found in skeletal muscle and heart (not specific for heart). LDH1 – heart muscle; LDH2 – blood serum. LDH levels are also high in tissue breakdown or hemolysis. Although CK-MB is more specific and sensitive for infarction than LDH. Aspartate transaminase is not specific for heart damage alone but can still be used to diagnose myocardial infarction. But troponin test [cardiac troponins I (cTnI) and T (cTnT)] is the most sensitive and specific test for myocardial infarction.

12. According to the data of WHO, for about 250 mln of Earth population fall ill with malaria. This disease is mostly spread in tropical and subtropical regions. Range of its spread falls into the areal of the following mosquitoes:

Explanation

13.

Autopsy of a 58 y.o. man revealed that bicuspid valve was deformed, thickened and unclosed. Microscopically: foci of collagen fibrilla are eosinophilic, react positively to fibrin. The most probably it is:

Explanation

        Due to the positive fibrin reaction and eosinophilia of the collagen fibers, it is better to choose fibrinoid swelling since there is no record of any inflammatory reaction or cells. Fibrinoid swelling changes is a stromal vascular proteinous degeneration. The main signs are revealed microscopically: the bands of collagen fibers are homogenous, impregnated with plasma proteins.

14.

Labeled amino acids alanine and tryptophane were introduced to a mouse in order to study localization of protein biosynthesis in its cells. Around what organellas will the accumulation of labeled amino acids be observed?

Explanation

A large number of components are required for translation (synthesis of a protein). These include all the amino acids that are found in the finished product, the mRNA to be translated, tRNA, functional ribosomes, energy sources and enzymes as well as protein factors needed for initiation, elongation and termination of the polypeptide chain. In eukaryotic cells, the ribosomes are either “free” in the cytosol or are in close association with the endoplasmic reticulum (which is then known as the “rough” endoplasmic reticulum or RER). The RER – associated ribosomes are responsible for synthesizing proteins that are to be exported from the cell as well as those that are destined to become integrated into plasma, endoplasmic reticulum or golgi membranes or incorporated into lysosomes.

15. In some regions of South Africa there is a spread sickle-shaped cell anemia, in which erythrocytes have shape of a sickle as a result of substitution of glutamin by valine in the hemoglobin molecule. What is the cause of this disease?

Explanation

    Sickle cell hemoglobin (HbS) point mutation causes a single amino acid replacement in β chain of hemoglobin (substitution of glutamic acid with valine). Pathogenesis of sickle cell anemia: low O2, high altitude or acidosis precipitates sickling – modified shape (deoxygenated HbS polymerizes) → anemia and vaso-occlusive disease. Clinical findings include: dizziness, general weakness, fatigue, “crew-cut” on skull X-ray due to marrow expansion from increased erythropoiesis (this is also seen in thalassemia)  etc. Sickle cells are crescent-shaped RBCs.

16.

A patient died under conditions of cardiovascular insufficiency. Autopsy results: postinfarction cardiosclerosis, myocardium hypertrophy and dilatation of its cavities, especially of its right ventricle. Liver is enlarged, its surface is smooth, incision revealed that it was plethoric, with dark-red specks against the background of brownish tissue. Histologically: plethora of central parts of lobules; peripheral parts around portal tracts contain hepatocytes in a state of adipose degeneration. How are these liver changes called?

Explanation

Chronic venous congestion of the liver occurs in right heart failure and sometimes due to occlusion of inferior vena cava and hepatic vein. The liver is enlarged and tender and the capsule is tense. Cut surface shows characteristic “nutmeg liver” due to red and yellow mottled appearance. The changes of congestion are more marked in the centrolobular zone due to severe hypoxia than in the peripheral zone. The peripheral zone of the lobule is less severely affected by chronic hypoxia and shows some fatty changes in the hepatocytes.

17. Autopsy of a man who died from chronic cardiovascular collapse revealed \\\"tiger heart\\\". Sidewards of endocardium a yellowish-white banding can be seen; myocardium is dull, dark-yellow. What process caused this pathology?

Explanation

Intracellular accumulations (parenchymal degeneration or dystrophies). Intracellular fatty degenerations are the abnormal accumulations of triglycerides within parenchymal cells. The liver, heart, kidneys are damaged the most frequently. Fatty degeneration of the heart – Tiger’s heart; fatty degeneration of the liver – Goose liver; fatty degeneration of the kidney – large white kidney.
18. In course of an experiment a skeletal muscle is being stimulated by a series of electric impulses. What type of muscle contraction will arise, if every subsequent impulse comes in the period of shortening of the previous single muscle contraction?

Explanation

Tetanus is defined as the sustained contraction of muscle due to repeated stimuli with high frequency. When the multiple stimuli are applied at a higher frequency in such a way that the successive stimuli fall during contraction period (period of shortening) of previous twitch, the muscle remains in state of tetanus. While increasing the frequency, fusion of contractions increases every time and finally complete tetanus occurs. Holotetanus – complete tetanus.

When the frequency of stimuli is not sufficient to cause tetanus, the fusion of contractions is not complete and then we have – incomplete/partial/waved tetanus. This occurs (i.e. the next stimuli occurs) during the period  of relaxation.

19. Ammonia is a very toxic substance, especially for nervous system. What substance takes the most active part in ammonia detoxification in brain tissues?

Explanation

Substances absorbed into the bloodstream from the intestine pass through the liver, where toxins are normally removed. Many of these toxins (such as ammonia) are normal breakdown products of the digestion of protein.  Ammonia is produced by amino acid metabolism and intestinal urease-positive bacteria. In physiological conditions, it is mostly present as ammonium (NH4+) in serum. The urea or ornithine cycle, which is fully expressed in the liver exclusively, serves to converts NH4+ to urea prior to renal excretion and to maintain low serum concentrations. 

NH3 + α-ketoglutarate → Glutamate

α-ketoglutarate is used up which leads to:

·        ↑glutamate → ↑GABA (inhibitory neurotransmitter)

·        Inhibition of citric acid cycle/tricarboxylic acid cycle; this causes impairment of ATP formation.

·        Inhibition of metabolism of amino acids (impairment of transamination reactions).

 

NH3 + Glutamate → Glutamine

Glutamine is an amide of glutamic acid which provides a non-toxic storage and transport form of ammonia (NH3). Ammonia increase synthesis of glutamine in brain. Accumulation of glutamine in brain results in elevation of osmotic pressure in nervous cells leading to brain edema.

                                   

                                    NH3 + H+ → NH4+

In blood ammonia (NH3) is represented as ammonium ion (NH4+). Accumulation of ammonium ion impairs transport of ions (Na+, K+) through cell membranes and failure of transmission of nerve impulse.

             

20.

A man was admitted to the hospital on the 5th day of disease that manifested itself by jaundice, muscle aching, chill, nose bleedings. In course of laboratory diagnostics a bacteriologist performed dark-field microscopy of the patient’s blood drop. Name a causative agent of this disease:

Explanation

          Three genera of spirochetes cause human infection:

·        Treponema: causes syphilis and the nonveneral treponematoses. Morphology – thin, tight spirals

·        Borrelia: causes lyme disease and relapsing fever. Morphology – large and loosely coiled

·        Leptospira: causes leptospirosis. Morphology – thin, tight spirals

Leptospira is tightly coiled, fine spirochetes that are not stained with dyes but are seen by darkfield microscopy. Leptospira interrogans is the cause of leptospirosis. Human infection results when leptospira is ingested or pass through mucus membranes or skin. They circulate in the blood and multiply in various organs, producing fever and dysfunction of the liver (jaundice), kidney (uremia), lungs (hemorrhage) and CNS (aseptic meningitis). Leptospira C and S letters.

21.

A patient has a transverse disruption of spinal cord below the IV thoracic segment. What changes of respiration will it cause?

Explanation

22. 12 hours after an acute attack of retrosternal pain a patient presented a jump of aspartate aminotransferase activity in blood serum. What pathology is this deviation typical for?

Explanation

In laboratory diagnosis of acute myocardial infarction: Creatine kinase isoenzyme MB (CK-MB) appears within 4-8hrs, peaks at 24hrs and disappears within 1.5-3 days. Sensitivity and specificity is 95%. Reappearance of CK-MB after 3days – reinfarction. CK-MM – found in skeletal muscle and heart (not specific for heart). LDH1 – heart muscle; LDH2 – blood serum. LDH levels are also high in tissue breakdown or hemolysis. Although CK-MB is more specific and sensitive for infarction than LDH. Aspartate transaminase is not specific for heart damage alone but can still be used to diagnose myocardial infarction. But troponin test [cardiac troponins I (cTnI) and T (cTnT)] is the most sensitive and specific test for myocardial infarction.

23.

A lightly dressed man is standing in a room, air temperature is +140C, windows and doors are closed. In what way does he emit heat the most actively?

Explanation

Heat Radiation is a way the surface of the human body emits heat to the environment in the form of infrared rays. The amount of heat the body radiates to the environment is proportional to the surface of radiation area and to the difference between the mean values of skin and environment temperature. The surface radiation area is the total surface area of body parts that contact the air. Elimination of heat by radiation increases with a decrease in ambient temperature and decreases with its increase. It is possible to reduce elimination of heat by radiation via reduction of the surface of radiation area (“winding oneself into a ball”). Heat radiation does not require a medium for transfer of heat. (Key words: naked or lightly clothed).

Convection is a way the body eliminates heat by means of transferring heat via moving particles of air or water. To dissipate heat by means of convection, body surface shall be airflowed at a temperature that is lower than the temperature of the skin. At that, air layer contacting with the skin warms up, decreases its density, rises and is replaced by cooler, denser air. By increasing the speed of the air flow (wind, ventilation) heat emission increases significantly as well (forced convection). Convection requires convection current; current of gases or liquids (Key words: air over exposed area of skin).

Evaporation is a way the body dissipates heat to the environment by its evaporation via sweat or evaporation of moisture from the skin and respiratory tract mucous membranes of (“wet” heat loss). Evaporation closely related to relative humidity.

Conduction is a way the body eliminates heat by means of direct contact with another object. Heat is transferred down the temperature gradient (i.e. from the object of higher temperature to the object of lower temperature). Conduction requires contact with another object (Key words:  in water).

24. A 30 y.o. woman had been ill for a year when she felt pain in the area of joints for the first time, they got swollen and skin above them became reddened. Provisional diagnosis is rheumatoid arthritis. One of the most probable causes of this disease is a structure alteration of a connective tissue protein:

Explanation

       Rheumatoid (atrophic) arthritis is a chronic progressive inflammatory arthritis of unknown origin involving multiple joints and characterized by disorganization of connective tissue (collagen) of the synovial membrane and articular cartilage and development of their deformation. Disorganization of the connective tissue cause increase in proteoglycans and glycosaminoglycans (GAGs) concentration in blood. Proteoglycans and GAGs are responsible for the physical properties of ground substance. Main morphological appearance of rheumatoid arthritis is synovitis. It has 3 stages: in the first stage, it is characterized by an acute inflammatory reaction with development of edema, hyperemia and infiltration of lymphocytes, plasma cells and macrophages. Small areas of superficial necrosis or superficial erosions are covered by fibrinoid deposits. Not infrequently 2-3mm “rice bodies” (rice grains) composed of fibrin, fibronectin, collagen and immunoglobulin are present in joint cavities of seropositive patients.

Collagen is a major component of the connective tissue.

25.

A patient with infectious mononucleosis has been taking glucocorticoids for two weeks. He was brought into remission, but he fell ill with acute attack of chronic tonsillitis. What action of glucocorticoids caused this complication?

Explanation

Glucocorticoids have immunosuppressive effects, anti-inflammatory effects, anti-shock, anti-allergic and anti-toxic effects. Immunosuppressive effects: glucocorticoids inhibit some of the mechanisms involved in cell-mediated immunologic functions, especially those dependent on lymphocytes. These agents are actively lymphotoxic and are important in the treatment of hematologic cancers. The drugs do not interfere with the development of normal acquired immunity but delay rejection reactions in patients with organ transplants. This immunosuppressive effects makes the patient susceptible to other infectious diseases (e.g. chronic tonsillitis).

26.

A peripheral segment of vagus nerve on a dog’s neck was being stimulated in course of an experiment. The following changes of cardiac activity could be meanwhile observed:

Explanation

Vagus nerve branches that innervate the heart produce parasympathetic effects on heart’s function e.g. ↓heart rate, force of contraction etc. This is as a result of hyperpolarization of the pacemaker cells. Hyperpolarization is as a result of K+ efflux (K+ yield) making it take a longer time for the cell to reach the threshold potential. This delay in reaching the threshold produces the delay or decrease in heart rate.

27. As a result of exhausting muscular work a worker has largely reduced buffer capacity of blood. What acidic substance that came to blood caused this phenomenon?

Explanation

Lactate, formed by the action of lactate dehydrogenase (converting pyruvate to lactate) is the final product of anaerobic glycolysis in eukaryotic cells.

Athletes that are exercising intensely for the short periods of time, such as in a sprint race, build up large amounts of lactate in their muscles as the result of anaerobic glycolysis. 

Increase lactate in blood can result in lactic acidosis, upsetting the fragile buffer system in the body.

28.

A patient was delivered to the hospital by an emergency team. Objectively: grave condition, unconscious, adynamy. Cutaneous surfaces are dry, eyes are sunken, face is cyanotic. There is tachycardia and smell of acetone from the mouth. Analysis results: blood glucose - 20,1 micromole/l (standard is 3,3-5,5 micromole/l), urine glucose - 3,5% (standard is - 0). What is the most probable diagnosis?

Explanation

     

29.

A 62 y.o. woman complains of frequent pains in the area of her chest and backbone, rib fractures. A doctor assumed myelomatosis (plasmocytoma). What of the following laboratory characteristics will be of the greatest diagnostical importance?

Explanation

Myeloma/Plasmocytoma/Multiple Myeloma

     Paraprotein, myeloma protein, M protein or spike protein is an abnormal immunoglobulin (Ig) fragment or immunoglobulin (Ig) light chain that is produced in excess by an abnormal clonal proliferation of plasma cells, typically in multiple myeloma. Monoclonal free light chains in the serum or urine are called bence jones (BJ) proteins.

Bence jones (BJ) protein: free kappa (κ) or lambda (λ) light chains that are excreted in urine associated with plasma cell malignancies (myeloma) and Waldenstrὄm macroglobulinemia. In myeloma, urinalysis for BJ protein is positive in 60-80% of cases.

30. Nappies of a newborn have dark spots that witness of formation of homogentisic acid. Metabolic imbalance of which substance is it connected with?

Explanation

Congenital deficiency of homogentisate oxidase (homogentisic acid oxidase) in the degradative pathway of tyrosine to Fumarate → pigment-forming homogentisic acid accumulates (homogentisuria) in tissues. Autosomal recessive. Usually benign. Urine turns black on prolonged exposure to air. May have debilitating arthralgias (homogentisic acid toxic to cartilage).
31.

Having helped to eliminate consequences of a failure at a nuclear power plant, a worker got an irradiation doze of 500 roentgen. He complains of headache, nausea, dizziness. What changes in leukocytes quantity can be expected 10 hours after irradiation?

Explanation

The nuclear radiation this patient was exposed to can produce inflammatory reaction in the patient. Within the first 24hours, the first leukocyte to get into any inflammatory focus are the neutrophils, followed by the monocytes (macrophages) and then lymphocytes.

Neutrophils are the first leukocytes that cross the blood vessel wall to enter inflammatory sites. Under normal conditions, leukocytes are restricted to the center of small blood vessels, where the flow is fastest. In inflammatory sites, where the vessels are dilated, the slower blood flow allows the leukocytes to move out of the center of the blood vessel and interact with the vascular endothelium. Even in the absence of infection, monocytes migrate continuously into the tissues, where they differentiate into macrophages; meanwhile, during an inflammatory response, the induction of adhesion molecules on the endothelial cells, as well as induced changes in the adhesion molecules expressed on leukocytes recruit large numbers of circulating leukocytes, initially Neutrophils and later monocytes, into the site of an infection (inflammatory focus).

First –Neutrophils; second –monocytes and macrophages; third –lymphocytes.

32. A 12 y.o. boy who suffers from bronchial asthma has an acute attack of asthma: evident expiratory dyspnea, skin pallor. What type of alveolar ventilation disturbance is it?

Explanation

Obstructive respiratory disease is the abnormal respiratory condition characterized by difficulty in expiration. E.g bronchial asthma, chronic bronchitis, emphysema, cystic fibrosis.                                           Restrictive respiratory disease is the abnormal respiratory condition characterized by difficulty in inspiration. E.g poliomyelitis,myasthenia gravis, paralysis of diaphragm, spiral cord diseases, pleural effusion, fibrosis. (lung fibrosis-pneumofibrosis)
33.

A woman who has been keeping to a clean-rice diet for a long time was diagnosed with polyneuritis (beri-beri). What vitamin deficit results in development of this disease?

Explanation

        Thiamine (vitamin B1): thiamine pyrophosphate (TPP) is the biologically active form of the vitamin, formed by the transfer of a pyrophosphate group from ATP to thiamine. Biological role of TPP: it is a component of pyruvate dehydrogenase and α-ketoglutarate dehydrogenase complexes catalyzing the reactions of oxidative decarboxylation of pyruvate and α-ketoglutarate (kreb’s cycle) i.e. it promotes energy formation from carbohydrates and lipids. It’s also a component of transketolase (pentose phosphate pathway of glucose oxidation) essential for fats and nucleic acids synthesis. Deficiency of Vit B1 causes Beri-Beri.

34.

Removal of gall bladder of a patient has disturbed processes of Ca absorption through the intestinal wall. What vitamin will stimulate this process?

Explanation

Parathyroid hormone: secreted by chief cells of parathyroid gland. Effects include:

↑bone resorption of Ca2+ and PO43- → ↑their plasma levels

↑kidney reabsorption of Ca2+ in distal convoluted tubule → ↑ Ca2+ plasma level

↓reabsorption of PO43- in proximal convoluted tubule → ↓ PO43- plasma levels

↑Calcitriol (vit D3) production by stimulating kidney 1α-hydroxylase in proximal convoluted tubule. It increases Ca2+ and PO43- absorption in the intestine.

In general, parathyroid hormone ↑ Ca2+ plasma level but ↓ PO43- plasma levels. Abnormal synthesis (↑synthesis) of parathyroid hormone can lead to hypercalcemia and hypophosphatemia.

Calcitonin is secreted by parafollicular cells (C cells of the thyroid gland). It ↓bone resorption of Ca2+. It opposes actions of parathyroid hormone. But its not important in normal Ca2+ homeostasis. Calcitriol ↑ circulating Ca2+ ions as a means of enhancing intestinal absorption of calcium (NB: Calcitriol production is dependent on parathyroid hormone).
35.

A hepatitis outbreak was registered in a settlement. This episode is connected with water factor. What hepatitis virus could have caused the infective outbreak in this settlement?

Explanation

Hepatitis E: incubation period 15 - 45days; it is an RNA-virus; transmission is fecal-oral especially waterborne. It is not transmitted by blood or sexually. It mostly occurs in developing countries. It produces acute hepatitis and fulminant hepatitis may develop in pregnant women.

36.

Examination of a 43 y.o. patient revealed that his stomach has difficulties with digestion of protein food. Gastric juice analysis revealed low acidity. Function of which gastric cells is disturbed in this case?

Explanation

Parietal (oxyntic) cells are found in the neck of the fundic glands, among the mucus neck cells and in the deeper part of the gland. They secrete HCl and intrinsic factor. When examined with the transmission electron microscope they are seen to have an extensive intracellular canalicular system that communicates with the lumen of the gland. Also an elaborate tubulo-vesicular membrane system is present in the cytoplasm adjacent to the canaliculi. Numerous mitochondria with complex cristae and many matrix granules supply the high levels of energy necessary for acid secretion.

37.

A 16 y.o. boy from a countryside entered an educational establishment. Scheduled Manteux test revealed that the boy had negative reaction. What are the most reasonable actions in this case?

Explanation

Tuberculin (Mantoux) skin test: this test is done by intradermal injection of tuberculoprotein (tuberculin), purified protein derivative (PPD). Type IV hypersensitivity reaction.

Immunization against tuberculosis is induced by injection of attenuated strains of bovine type of tubercle bacilli, Bacilli Calmette Guerin (BCG).

The Mantoux skin test should be read between 48 and 72hrs after administration. The basis of reading  is the presence or absence of induration, which may be determined by inspection and by palpation. A record should also be made of formation of vesicles, bullae, lymphangitis, ulceration and necrosis at the test site. The formation of vesicles, bullae or necrosis at the test site indicates positive result. A negative mantoux result usually signifies that the individual has never been exposed to Mycobacterium tuberculosis i.e. absence of cell mediated immunity to tuberculin; which is an indication for immunization with BCG vaccine.

38. A patient complains of frequent diarrheas, especially after consumption of fattening food, and of body weight loss. Laboratory examination revealed steatorrhea; hypocholic feces. What can be the cause of this condition?

Explanation

Obturation (obstruction, to close) of bile duct – it can be:

* Intrahepatic – blockage of intrahepatic bile ducts

* Extrahepatic – blockage of common bile duct (ductus choledochus).

Findings:

* malabsorption: bile salts do not enter the Small Intestine; no emulsification of fat.

*light coloured stool: due to lack of urobilin (which leads to lack of stercobilin).

*Jaundice (posthepatic, mechanic, obstructive): increased conjugated Bilirubin.

* Steatorrhea

The findings are specific for obstruction of bile duct and bile acid deficiency.

39.

A patient consulted a doctor about bowels dysfunction. The doctor established symptoms of duodenitis and enteritis. Laboratory examination helped to make the following diagnosis: lambliosis. What medication should be administered?

Explanation

Metronidazole forms toxic free radical metabolites in the bacterial cell that damage DNA. It is bactericidal, and an antiprotozoal. It is used to treat Giardia, Entamoeba, Trichomonas, Gardnerella vaginalis, Anaerobes (bacteroides, Clostridium difficile). It can be used with a proton pump inhibitor and clarithromycin for “triple therapy” against Helicobacter pylori. NB: Giardia lamblia is the causative agent of Lambliosis.

40. A group of mountain climbers went through the blood analysis at the height of 3000 m. It revealed decrease of HCO3 to 15 micromole/l (standard is 22-26 micromole/l). What is the mechanism of HCO3 decrease?

Explanation

41.

Introduction of a pharmaceutical substance to an experimental animal resulted in reduction of salivation, pupil mydriasis. Next intravenous introduction of acetylcholine didn’t lead to any significant changes of heart rate. Name this substance:

Explanation

Acetylcholine is a neurotransmitter. It increases salivary secretion and stimulates intestinal secretions and motility. It increases bronchial secretions, increases tone of detrusor urinae muscle, causing expulsion of urine, stimulates ciliary muscle contraction (accommodation). It increases exocrine gland secretions (e.g. lacrimal, salivary, gastric acid); gut peristalsis, bladder contraction, bronchoconstriction, papillary sphincter muscle contraction (miosis). Poisoning by cholinomimetics causes diarrhea, urination, miosis, bronchospasm, bradycardia, excitation of skeletal muscle and central nervous system, lacrimation, sweating and salivation. Cholinomimetics are cholinergic agonists which produce analogous effects of acetylcholine.

Atropine sulfate (M-cholinoblocker) blocks the acetylcholine response, so that intravenous introduction of acetylcholine didn’t lead to any significant changes. 
42.

Continuous taking of a drug can result in osteoporosis, erosion of stomach mucous membrane, hypokaliemia, retention of sodium and water, reduced content of corticotropin in blood. Name this drug:

Explanation

Osteoporosis is a common adverse effect of long-term corticosteroid therapy, due to the ability of glucocorticoids (prednisolone) to suppress intestinal Ca2+ absorption, inhibit bone formation and decrease sex hormone synthesis. Other adverse effects include peptic ulcer (erosions of stomach mucosa), hypokalemia, emotional disturbances and the classic cushing-like syndrome (i.e. redistribution of body fat, puffy face, increased body hair growth, acne, insomnia and increased appetite) are observed when excess corticosteroids are present. Excess corticosteroid results in decreased corticotrophin (adrenocorticotropic hormone) through the negative feedback mechanism.

43. A child is languid, apathetic. Liver is enlarged and liver biopsy revealed a significant excess of glycogen. Glucose concentration in the blood stream is below normal. What is the cause of low glucose concentration?

Explanation

          Glycogen phosphorylase is the rate-determining enzyme in Glycogenolysis (break down of glycogen). Glycogen phosphorylase cleaves glycogen to glucose 1-phosphate (first step in glycogenolysis). This pathway mobilizes stored glycogen in liver to replenish used glucose. It also breaks down glycogen in muscle to glucose, to produce energy during physical work. If this enzyme is deficient, then used glucose cannot be replaced leading to hypoglycemia and related symptoms.

In lysosomal glycosidase, there is still normal blood sugar levels (no hypoglycemia). α-amylase and ɣ-amylase is involved in digestion of polysaccharides. Glucose 6-phosphate dehydrogenase (G6PD) is the rate-limiting enzyme in pentose phosphate pathway which catalyzes an irreversible oxidation of glucose 6-phosphate to 6-phosphogluconolactone.

44. A 63 y.o. man with collapse symptoms was delivered to the emergency hospital. A doctor chose noradrenaline in order to prevent hypotension. What is the action mechanism of this medication?

Explanation

       Because norepinephrine is the neuromediator of adrenergic nerves, it should theoretically stimulate all types of adrenergic receptors. In practice, when the drug is given in therapeutic doses to humans, the α-adrenergic receptor is most affected. Norepinephrine causes a rise in peripheral resistance due to intense vasoconstriction of most vascular beds, including the kidney (α1 effect). Both systolic and diastolic blood pressures increase. α1 receptors are present on the postsynaptic membrane of the effector organs and mediate constriction of smooth muscle. NB: norepinephrine (noradrenaline) causes greater vasoconstriction than epinephrine, because it does not induce compensatory vasodilation via β2 receptors on blood vessels supplying skeletal muscles.

45.

From pharynx of a child with suspected diphtheria a pure culture of microorganisms was isolated. Their morphological, tinctorial, cultural and biochemical properties appeared to be typical for diphtheria causative agents. What study should be conducted in order to draw a conclusion that this is a pathogenic diphtheria bacillus?

Explanation

Having carried out all these examinations (morphological, tinctural, cultural and biochemical), then to draw a conclusion on its pathogenicity (i.e. its ability to cause disease), another examination determining the toxigenicity of this organism needs to be performed. Because the exotoxins produced by corynebacterium diphtheria cause the disease manifestations. Exotoxin production is essential for pathogenesis. Diphtheris toxin inhibits protein synthesis by ADP-ribosylation of elongation factor 2 (EF-2). The toxin affects all eukaryotic cells.

46.

Examination of a child revealed some whitish spots looking like coagulated milk on the mucous membrane of his cheeks and tongue. Analysis of smears revealed gram-positive oval yeast-like cells. What causative agents are they?

Explanation

         There are two types of fungi: yeasts and molds. Yeasts grow as single cells that reproduce by asexual budding (gemmating). Molds grow as long filaments (hyphae) and form a mat (mycelium). Candida albicans is an oval yeast with a single bud. It is part of the normal flora of mouth (oral cavity), mucous membranes of upper respiratory, gastrointestinal and female genital tracts. In tissues it may appear as yeast or as pseudohyphae. Pseudohyphae are elongated yeasts (elongated chains) that visually resemble hyphae but are not true hyphae. Candida albicans causes thrush, vaginitis, esophagitis and chronic mucocutaneous candidiasis.

47. A 35 y.o. patient who often consumes alcohol was treated with diuretics. There appeared serious muscle and heart weakness, vomiting, diarrhea, AP- 100/60 mm Hg, depression. This condition is caused by intensified excretion with urine of:

Explanation

Drugs inducing a state of increased urine flow are called diuretics. These agents are inhibitors of renal ion transporters that decrease the reabsorption of Na+ (increase excretion) and increase K+ excretion at different sites in the nephron. All diuretics (thiazides, loop diuretics, carbonic anhyrase inhibitors and osmotic diuretics) are known to cause potassium depletion (hypokalemia) except the potassium sparing diuretics (spironolactone, triamterene). Skeletal muscle and myocardial weakness, vomiting, diarrhea are common signs of hypokalemia.
48.

After intake of rich food a patient feels nausea and sluggishness; with time there appeared signs of steatorrhea. Blood cholesterine concentration is 9,2 micromole/l. This condition was caused by lack of:

Explanation

Steatorrhea is the formation of bulky, foulsmelling, frothy and clay-coloured stools with large quantity of undigested fat because of impaired digestion and absorption of fat. Any condition that causes indigestion or malabsorption of fat leads to steatorrhea. Causes of steatorrhea are: lack of pancreatic lipase, liver disease affecting the secretion of bile (lack of bile acid or obstruction of bile duct), celiac disease and cystic fibrosis.

49.

Examination of a man who hadn’t been consuming fats but had been getting enough carbohydrates and proteins for a long time revealed dermatitis, poor wound healing, vision impairment. What is the probable cause of metabolic disorder?

Explanation

Lipid malabsorption resulting in increased lipid (including the fat soluble vitamins A, D, E, K and essential fatty acids – linoleic acid) in the faeces (steatorrhea), can be caused by disturbances in lipid intake or digestion and/or absorption. Lack of vitamin A (vision impairment and poor wound healing); lack of linoleic acid (dermatitis).

50. An experimental animal has been given excessive amount of carbon-labeled glucose for a week. What compound can the label be found in?

Explanation

In adults, fatty acid synthesis occurs primarily in the liver and lactating mammary glands and to a lesser extent in adipose tissue. The process incorporates carbons from acetyl CoA into the growing fatty acid chain, using ATP and reduced NADPH. The energy for the carbon-to-carbon condensations in fatty acid synthesis is supplied by the process of carboxylation and then decarboxylation of acetyl groups in the cytosol. Palmitic acid (palmitate), a 16-carbon, fully saturated long chain length fatty acid is the primary end product of fatty acid synthase activity.

51.

RNA that contains AIDS virus penetrated into a leukocyte and by means of reverse transcriptase forced a cell to synthesize a viral DNA. This process is based upon:

Explanation

RNA dependent DNA polymerase

     DNA polymerase – synthesizes DNA

     RNA dependent – from mRNA (on the matrix of virus mRNA)

        Human immunodeficiency virus (HIV): diploid genome (2 molecules of RNA). The 3 structural genes (i.e. proteins coded for by the genes) are:

·        env (gp 120 and gp 41): formed from cleavage of gp 160 to form envelope glycoproteins. gp 120 is for attachment to host CD4+ T cell. gp 41 is for fusion and entry.

·        Gag (p24): capsid protein

·        pol: reverse transcriptase, aspartate protease, integrase.

ELISA/Western blot (immunoblot) tests look for antibodies to the viral proteins listed above.

Reverse transcriptase synthesizes dsDNA (ds-double stranded) from genomic RNA (mRNA); dsDNA integrates into host genome in a process called reverse transcription. Virus binds CD4 as well as a coreceptor, either CCR5 on macrophages (early infection) or CXCR4 on I cells (late infection).

·        Homozygous CCR5 mutation – immunity

·        Heterozygous CCR5 mutation – slower course.

52. Examination of a patient with frequent hemorrhages from internals and mucous membranes revealed proline and lysine being a part of collagen fibers. What vitamin absence caused disturbance of their hydroxylation?

Explanation

Vitamin C (ascorbic acid): found in fruits and vegetables; an antioxidant; also facilitates iron absorption by reducing it to Fe2+ state. It is necessary for hydroxylation of proline and lysine in collagen synthesis; necessary for dopamine β-hydroxylase, which converts dopamine to norepinephrine. Deficiency leads to: scurvy – swollen gums, bruising, petechiae, hemarthrosis, anemia, poor wound healing, perifollicular and subperiosteal hemorrhages, “corkscrew” hair; Weakened immune response.

Type III collagen is found in blood vessels; Type IV collagen is found in basement membrane. Deficiency in Vitamin C disrupts the second stage of collagen synthesis in fibroblasts (hydroxylation of collagen) which results in petechiae, bruising, hemarthrosis.

Vitamin B2 (riboflavin) deficiency – growth retardation, glossitis, conjunctivitis

Vitamin B1 (thiamine) deficiency – Beri-Beri (polyneuritis)

Vitamin A (retinol) deficiency – Night blindness

53. A patient who suffers from pneumonia has high body temperature. What biologically active substance plays the leading part in origin of this phenomenon?

Explanation

     The causative agent of the Pneumonia causes the release of pyrogens that activates the immune system. Pyrogenic cytokines, principally IL-1; IL-6 and TNF-α are released into the bloodstream for transport to the hypothalamus, where they exert their action. These cytokines induce prostaglandin E2 (PGE2), which is a metabolite of arachidonic acid. It is hypothesized that when IL- 1β interacts with the endothelial cells of the blood-brain barrier in the capillaries of the organum vasculosum laminae terminalis (OVLT), which is in the third ventricle above the optic chiasm, PGE2 is released into the hypothalamus. At this point PGE2 binds to receptors in the hypothalamus to induce increase in the thermostatic set point through the second messenger cAMP. In response to the increase in its thermostatic set point, the hypothalamus initiates shivering and vasoconstriction that raise the body’s core temperature to the new set point and fever is established.

54.

A 27 y.o. patient put eye drops that contain penicillin. After a few minutes she felt itching and burning of her body, there appeared lip and eye-lid edemata; arterial pressure began to drop. What immunoglobulins took part in the development of this allergic reaction?

Explanation

Type I Hypersensitivity reaction (HSR); anaphylactic and atopic: free antigen cross-links IgE on presensitized (i.e. exposed to the antigen before) mast cells and basophils, triggering immediate release of vasoactive amines that act at postcapillary venules (i.e. histamine). Reaction develops rapidly after antigen exposure because of preformed antibody from first exposure. IgE is the main immunoglobulin involved in type I HSR. Type I: uses IgE and IgG4

55. A patient suffers from hepatic cirrhosis. Examination of which of the following substances excreted by urine can characterize the state of antitoxic function of liver?

Explanation

Hippuric acid has been a major human metabolite for years. However, there is no well-known documented health benefit associated with it except for excretion of environmental-toxic exposures of aromatic compounds such as toluene or from dietary protein degradation and resynthesis by intestinal microflora metabolism of quinic acid via the shikimate pathway. Thus hippuric acid can appear in humans as an excretory product from natural or unnatural sources. It has been believed over the years that the major source of urinary hippuric acid levels in humans has come from environmental toxic solvent exposures.
56.

 A 2 y.o. child has convulsions as a result of lowered concentration of calcium ions in blood plasma. It is caused by reduced function of:

Explanation

Parathyroid hormone: secreted by chief cells of parathyroid gland. Effects include:

↑bone resorption of Ca2+ and PO43- → ↑their plasma levels

↑kidney reabsorption of Ca2+ in distal convoluted tubule → ↑ Ca2+ plasma level

↓reabsorption of PO43- in proximal convoluted tubule → ↓ PO43- plasma levels

↑Calcitriol (vit D3) production by stimulating kidney 1α-hydroxylase in proximal convoluted tubule. It increases Ca2+ and PO43- absorption in the intestine.

In general, parathyroid hormone ↑ Ca2+ plasma level but ↓ PO43- plasma levels. Reduction of parathyroid hormone can lead to hypocalcemia (lowered concentration of calcium in blood) and hyperphosphatemia.

57.

  Inflammation of a patient’s eye was accompanied by accumulation of turbid liquid with high protein at the bottom of anterior chamber that was called hypopyon. What process underlies the changes under observation?

Explanation

The aqueous humor is continuously produced by the capillaries of the ciliary processes. The humor fills the zonular spaces from where it proceeds to the chambers. The humor drains to the blood stream via the spaces of iridocorneal angle and further to the sclera venous sinus. The sinus communicates with the sclera veins. This process ensures constant pressure (ocular tension) within the eyeball (in order to maintain a certain shape). Drainage disorders in this microcirculatory circuit can lead to accumulation of fluid in the eye chambers. Hypopyon is inflammatory cells in the anterior chamber of the eye.

58. Heart rate of a man permanently equals 40 beats pro minute. What is the pacemaker?

Explanation

59.

A 48 y.o. patient was admitted to the hospital with complaints about weakness, irritability, sleep disturbance. Objectively: skin and scleras are yellow. In blood: conjugated bilirubin, cholalemia. Feces are acholic. Urine is of dark colour (bilirubin). What jaundice is it?

Explanation

 

Indirect; Hemolytic; Prehepatic

Mixed; Parenchymal; Hepatic

Direct; Obstructive; Mechanic; Posthepatic

Stercobilin (faeces)

        ↑↑↑

Decreases (pale faces)

Absent (clay coloured faeces)

Type of bilirubin in blood

Unconjugated

Conjugated and Unconjugated

Conjugated

Obturation (obstruction, to close) of bile duct – it can be:

* Intrahepatic – blockage of intrahepatic bile ducts

* Extrahepatic – blockage of common bile duct (ductus choledochus).

Findings:

* malabsorption: bile salts do not enter the Small Intestine; no emulsification of fat.

*light coloured stool: due to lack of urobilin (which leads to lack of stercobilin).

*Jaundice (posthepatic, mechanic, obstructive): increased conjugated Bilirubin.

* Steatorrhea

The findings are specific for obstruction of bile duct and bile acid deficiency.

60.

Bacteriological examination of a patient with food poisoning required inoculation of a pure culture of bacteria with the following properties: gram-negative movable bacillus that grows in the Endo’s medium in form of colourless colonies. A representative of which species caused this disease?

Explanation

Both Endo Agar and EMB agar (Eosin-Methylene-Blue-Agar) are selective stain for Gram–ve bacteria, growth of gram+ve  organisms is inhibited. They provide a color indicator distinguishing between organisms that ferment lactose (e.g E.coli) and those that do not (e.g salmonella). Lactose fermenters (e.g E.coli) give a green metallic sheen. Lactose non-fermenters produce clear, colourless colonies.  

61.

A patient was diagnosed with active focal pulmonary tuberculosis. What drug should be prescribed in the first place?

Explanation

Isoniazid is the hydrazide of isonicotinic acid and is a pyridine. Pyridine occurs in many important compounds including  azines and the Vitamins Niacin(B3) and Pyridoxine(B6). Therefore, isoniazid can interfere with Vit.B3,B6 and even B1 metabolism by competing with them.

Isoniazid is a first line anti-tuberculosis drug that inhibits the synthesis of mycolic acid. Vit B6 is needed for the transformation of tryptophan to Vit.B3.

Adverse reaction of Isoniazid: peripheral neuritis, optic neuritis, hepatitis and idiosyncratic hepatotoxicity.
62.

Examination of a young man in the AIDS centre produced a positive result of immune-enzyme assay with HIV antigens. Patient’s complaints about state of his health were absent. What can the positive result of immune-enzyme assay be evidence of?

Explanation

Positive result of immune-enzyme assay with HIV antigens is an evidence of a current HIV infection.

63.

Parents of a 10 y.o. boy consulted a doctor about extension of hair-covering, growth of beard and moustache, low voice. Intensified secretion of which hormone must be assumed?

Explanation

The clinical manifestations are secondary sexual characteristics of an adult male.     

Early in male development, mesenchyme separating the seminiferous cords gives rise to leydig (interstitial) cells that produce testosterone to stimulate development of the indifferent primordium into a testis. Leydig cells are large, polygonal, eosinophilic cells that typically contain lipid droplets. Leydig cells differentiate and secrete testosterone during early fetal life. Secretion of testosterone is required during embryonic development, sexual maturation and reproductive function:

·        In the embryo, secretion of testosterone and other androgens is essential for the normal development of the gonads in the male fetus.

·        At puberty, secretion of testosterone is responsible for the initiation of sperm production, accessory sex gland secretion and development of secondary sex characteristics.

·        In the adult, secretion of testosterone is essential for the maintenance of spermatogenesis and of secondary sex characteristics, genital excurrent ducts and accessory sex glands.

The manifestation of these characteristics in a 10year old boy is as a result of increased secretion of the male sex hormone – Testosterone.
64. A patient who suffers from severe disorder of water-salt metabolism experienced cardiac arrest in diastole. What is the most probable mechanism of cardiac arrest in diastole?

Explanation

Hyperkalemia (serum K+ >5mEq/L): causes include tissue breakdown; increased intake; transcellular shift (extracellular); decreased renal excretion. Clinical findings in hyperkalemia includes ventricular arrhythmias – severe hyperkalemia (e.g. 7 – 8mEq/L) causes the heart to stop in diastole. ECG shows peaked T waves due to accelerated repolarization of cardiac muscle; muscle weakness and depressed/absent deep tendon reflexes. Hyperkalemia partially depolarizes the cell membrane, which interferes with membrane excitability.

65.

Examination of coronary arteries revealed atherosclerotic calcific plaques that close vessel lumen by 1/3. The muscle has multiple whitish layers of connective tissue. What process was revealed in myocardium?

Explanation

Diffuse cardiosclerosis

Diffuse – multiple whitish layers of connective tissue

Cardiosclerosis – Atherosclerotic calcified plaques

Focal cardiosclerosis. Focal –white fibrous depressed area 3cm in diameter.    Cardiosclerosis - Atherosclerosis.
66. Reaction of passive hemagglutination conducted with erythrocytic typhoid Vi-diagnosticum helped to reveal some antibodies in the dilution of the patient’s serum at a ratio of 1:80 that exceeds the diagnostic titer. Such result witnesses of:

Explanation

When the dilution of the patient’s serum exceeds the diagnostic titer, the patient is a potential carrier of the infectious agent. If less than the diagnostic titer, the examination is carried out again in 10 – 14days after the first examination. A dilution of the patient’s serum that gives the diagnostic titer indicates an acute or current infection.

67. A teenager was irradiated with high radiation dose that resulted in serious damages of lymphoid system, lysis of many lymphocytes. Restoration of normal hemogram is possible due to the functioning of the following gland:  

Explanation

Hemogram is a complete blood count that contains all information required for assessment of hematopoiesis as well as visual assessment of plasma appearance. Hematopoiesis takes place in the bone marrow but T-lymphocytes leave the red bone marrow in immature forms and go to the thymus to mature and be stored. So the thymus can play a significant role in restoration of normal hemogram.

68. A man with cut wound of his right foot sole was admitted to the hospital ward. The patient has limited elevation of the lateral foot edge. In course of wound management the injury of a muscle tendon was revealed. What muscle is injured?

Explanation

Lateral group of muscles of the leg have a common origin. The muscles of the lateral group (long and short peroneal muscles – peroneus longus and brevis) originate from the lateral condyle of the tibia, head and body of the fibula as well as from the deep fascia of the leg and its intermuscular septa. The peroneus longus (long peroneal muscle) resides superficially. It belongs to the pinnate muscles. In the lower third, it continues into the long tendon. The tendon descends and runs behind the lateral malleolus crossing over with the tendon of the peroneus brevis. Further, the tendon changes its direction and extends to the plantar surface of the foot. Here, it lies in the groove of the cuboid bone and runs obliquely across the foot inserting on its medial side. The peroneus brevis (short peroneal) lies underneath the peroneus longus muscle directly on the fibula. Peroneus longus (long peroneal muscle) pronates, abducts and flexes the foot.

69. A patient who suffers from heart failure has enlarged liver, edemata of lower extremities, ascites. What is the leading mechanism in the development of this edema?

Explanation

The main symptoms of right-sided heart failure are fluid accumulation and swelling (edema) of the feet, ankles, legs, liver and abdomen. Where the fluid accumulates depends on the amount of excess fluid and the effects of gravity. If a person is standing, fluid accumulates in the legs and feet; if a person is lying down, fluid usually accumulates in the lower back; if the amount of fluid is large, fluid also accumulates in the abdomen.

Most water leakage occurs in capillaries or postcapillary venules which have a semipermeable membrane wall that allows water to pass freely than proteins. In cardiac insufficiency, blood pools in the veins since the heart is not pumping effectively; this increases the hydrostatic pressure in the veins. And this increased hydrostatic pressure leads to increase filtration of water into the interstitium at the venous end of the capillary (post-capillary venule).

Hydrostatic - Hydrodynamic
70. A 32 y.o. man is tall, he has gynecomastia, adult woman pattern of hair distribution, high voice, mental deficiency, sterility. Provisional diagnosis is Klinefelter’s syndrome. In order to specify diagnosis it is necessary to analyze:

Explanation

Barr body is an inactive X-chromosome. So a boy (XY) with an inactive X-chromosome must have an additional X-chromosome – XXY (Klinefelter’s syndrome). Causes :

* nondisjunction (maternal and paternal nondisjunction in meiosis I)

* Mosaicism: with the karyotype being 46, XY/47, XXY

Manifestations: gynecomastia, female pattern of pubic hair distribution, no facial hair, high voice.

Diagnosis of most numerical chromosomal disorders (down, patau, edward, klinefelter, turner's) can be carried out using the Karyotype.

71. Examination of a miner revealed pulmonary fibrosis accompanied by disturbance of alveolar ventilation. What is the main mechanism of this disturbance?

Explanation

Pulmonary fibrosis occurs when lung tissue becomes damaged and scarred. This thickened, stiff tissue makes it more difficult for the lungs to work properly. As pulmonary fibrosis worsens, the patient becomes progressively more short of breath as a result of reduced alveoli available for gaseous exchange (limitation of respiratory surface) because the alveoli serves as the main respiratory surface of the lungs.

72. A man took a quiet expiration. Name an air volume that is meanwhile contained in his lungs:  

Explanation

73. Examination of an isolated cardiomyocyte revealed that it didn’t generate excitation impulses automatically. This cardiomyocyte was obtained from:

Explanation

Sinoatrial node, Atrioventricular node, His’ bundle, purkinje fibers are all parts of the conductive system of the heart and are capable of generating excitation impulses automatically. The ventricles are not capable of generating excitation impulses they only contract in response to impulses. The ventricular myocardium will only generate impulses in pathological conditions in the presence of an ectopic foci.

74. Examination of a man established that cardiac output equaled 3500 ml, systolic output - 50 ml. What is the man’s heart rate pro minute?

Explanation

Cardiac output (C.O.) is the amount of blood pumped out from each ventricle. Systolic or stroke volume is the amount of blood pumped out by each ventricle during each heart beat.

C. O. = Systolic volume X Heart rate (HR)

3500 = 50 X HR

HR = 70bpm (beats per minute)

75. A man who went for a ride on a roundabout had amplification of heart rate, sweating and nausea. What receptors stimulation is it primarily connected with?

Explanation

        The receptor areas of the vestibular labyrinth are represented with the following structures:

·        Macula of Utricle

·        Macula of Saccule

·        Ampullary crests

The first two detect linear motion, while the ampullary crest detect angular motion. The vestibulocochlear nerve supply these receptors. Both maculae consist of the sensory hair cells covered with jelly-like susbstance. The substance contains the crystals of calcium carbonate called otoliths (vestibular otolith). The utricle and saccule detect linear movement, also contributing to balance. The ampullary crests reside within each membranous ampulla. They also comprise the sensory hair cells covered with the same jelly-like substance called the ampullary cupula. The ampullae is the sensory organ in the semicircular canal that sense angular (rotational) acceleration of the head, thereby regulating balance. 

76. In order to estimate toxigenity of diphtheria agents obtained from patients the cultures were inoculated on Petri dish with nutrient agar on either side of a filter paper strip that was put into the centre and moistened with antidiphtheric anti-toxic serum. After incubation of inoculations in agar the strip-like areas of medium turbidity were found between separate cultures and the strip of filter paper. What immunological reaction was conducted?

Explanation

          Precipitation in agar (Precipitation gel reaction): antibody is incorporated into agar and antigen is inoculated i.e. antibody is the antidiphtheric antitoxic serum and the antigen is the diphtheria agent, inoculated in the nutrient agar. Antigen and antibody are placed in different wells in agar and allowed to diffuse and form concentration gradients. Where optimal proportions occur, lines of precipitate form. This method indicates whether antigens are identical, related but not identical or not related.

Areas of medium turbidity indicate where the antibody specific for the antigen have cross-linked.

77. A man’s intrapleural pressure is being measured. In what phase did the man hold his breath, if his pressure is 7,5 cm Hg?

Explanation

7.5cmHg – Quiet Inspiration

25cmHg – Forced Inspiration

78. A 49 y.o. woman consulted a doctor about heightened fatigue and dyspnea during physical activity. ECG: heart rate is 50/min, PQ is extended, QRS is unchanged, P wave quantity exceeds quantity of QRS complexes. What type of arrhythmia does the patient have?

Explanation

Atrioventricular (AV) block is the heart block in which the impulses are not transmitted from atria to ventricles (through AV node) because of defective conductive system. AV block can be 1st degree, 2nd degree (Mobitz I and II) and 3rd degree heart block. In complete or 3rd degree heart block, the impulses produced by the sinoatrial (SA) node cannot get to the ventricles. Therefore, atrial P waves and ventricular complexes (QRS) are recorded independently of each other; the number of ventricular complexes is usually much smaller than the number of atrial (P) waves (i.e. the number of P waves exceeds the number of QRS complexes). The heart rate in persistent complete heart block may be sufficiently high (40 – 50 beats/min) but the patient may be unaware of the disease for a long time.

79. A patient with neuritis of femoral nerve has disturbed flexion of thigh as well as disturbed crus extension in the knee joint. What muscle’s function is disturbed?

Explanation

The muscles of the thigh are divided into the anterior, medial and posterior groups. The anterior group – flexors of the thigh; posterior group – extensors of the thigh and medial group – adductors. The quadriceps femoris is the strongest muscle of the thigh, which occupies the entire anterior, lateral and partially medial surfaces of the thigh. It consists of four heads: rectus femoris, vastus medialis, vastus intermedius and vastus lateralis. All four heads of the quadriceps femoris in the lower part of the thigh fuse together and continue as a common tendon. This muscle acts on the hip joint and participates in the flexion of the thigh; its also a powerful extensor of the leg at the knee joint. It is supplied by the femoral nerve.

80. Autopsy of a newborn boy revealed polydactylia, microcephalia, cheiloschisis and uranoschisis as well as hypertrophy of parenchymatous organs. These defects correspond with the description of Patau’s syndrome. What is the most probable cause of this pathology?

Explanation

Trisomy 13 (Patau’s syndrome): the main abnormalities of this syndrome are mental retardation, holoprosencephaly, congenital heart defects, microcephaly, polydactylyl, microphthalmus, anophthalmos, cleft lip and palate (cheiloschisis and uranoschisis respectively).

81. An ovary specimen stained by hematoxylin-eosin presents a follicle, where cells of follicular epithelium are placed in 1-2 layers and have cubic form, there is a bright-red membrane around the ovocyte. What follicle is it?

Explanation

At puberty, a pool of growing follicles is established and continuously maintained from the supply of primordial follicles. A primary oocyte, together with its surrounding flat epithelial cells forms the primordial follicle. Each month 15 – 20 follicles selected from this pool begin to mature, passing through 3 stages namely: primary or preantral; secondary or antral and preovulatory (graafian follicle). As the primary oocyte begins to grow, surrounding follicular cells change from flat to cuboidal and proliferate to produce a stratified epithelium (which can be 1 - 2 layers but usually more) of granulose cells and the unit is called a primary follicle.

82. A woman has been applying a new cosmetic preparation for a week that resulted in eye-lid inflammation accompanied by hyperemia, infiltration and painfulness. What type of allergic reaction was developed?

Explanation

Type IV (cell mediated, delayed): antibody-independent T-cell mediated reactions e.g. positive mantoux reaction (tuberculin test), hashimoto’s thyroiditis, transplant rejection,  Contact dermatitis, Graft-versus-host disease, multiple sclerosis etc. Response is usually delayed.

83. Examination of a patient revealed extremely myotic pupils, sleepiness, infrequent Chain-Stoke’s respiration, urinary retention, slowing-down of heart rate, enhancement of spinal reflexes. What substance caused the poisoning?

Explanation

84. A patient is followed up in an endocrinological dispensary on account of hyperthyreosis. Weight loss, tachycardia, finger tremor are accompanied by hypoxia symptoms - headache, fatigue, eye flicker. What mechanism of thyroid hormones action underlies the development of hypoxia?

Explanation

A peculiar variant of histotoxic (tissue) hypoxia arises during acute dissociation or uncoupling of processes of oxidation and phosphorylation reactions in respiratory chain. The agents that dissociate process of oxidation and phosphorylation are wide range of substances of exogenous and endogenous origin which includes: hormones of thyroid gland, excess calcium, toxins etc. In a healthy organism, thyroid hormones (thyroxine and triiodothyronine) carry out the function of physiologic regulation of the association of oxidation and phosphorylation degree, together with other functions.

85. A doctor administered a patient with allergic dermatitis a H1-histamine blocker as a part of complex treatment. Name this medication:

Explanation

       Loratadine is an antihistamine (antiallergic drug) which refers to the classic H1-receptor blocker. These compounds do not influence the formation or release of histamine. Rather, they block the receptor-mediated response of a target tissue. They are much more effective in preventing symptoms than reversing them once they have occurred. These agents, desloratadine, fexofenadine, and loratadine show the least sedation i.e. least effect on CNS (they don’t induce sleep).

86. An injured man has bleeding from branches  of  carotid  artery.  For a temporary arrest of bleeding it is necessary to press the carotid artery to the tubercle of a cervical vertebra. Which vertebra is it?

Explanation

For temporary control of hemorrhage by digital (finger) pressing, press on the artery at the sites where it lies superficially and around a bone:

·        Carotid artery: transverse process of the C6 vertebra

·        Subclavian artery: first rib

·        Brachial artery: internal surface of the humerus

·        Femoral artery: pubic bone

87. As a result of an accident a patient has intense painfullness and edema of the anterior crus surface; dorsal flexion of foot is hindered. Function of which crus muscle is most likely to be disturbed?

Explanation

The anterior group of muscles of the leg (crus – leg): tibialis anterior, extensor digitorum longus, extensor hallucis longus. They have a common origin which is the lateral condyle of the tibia, both bones of the leg, their interosseus membrane as well as from the deep fascia of the leg. The tibialis anterior muscle lies superficially. In the upper part, it has a fleshy muscular part, which continues into the long flat tendon in the lower third of the leg. The tendon bends medially and attaches to the bones on the plantar surface of the medial side of the foot. It dorsiflexes (dorsal flexion) the foot at the ankle and inverts the foot (supinates the foot). Function of tibialis anterior muscle is disturbed if the dorsal flexion of foot is hindered.

88. The first grade pupils were examined in order to sort out children for tuberculosis revacination. What test was applied for this purpose?

Explanation

Tuberculin (Mantoux) skin test: this test is done by intradermal injection of tuberculoprotein (tuberculin), purified protein derivative (PPD). Type IV hypersensitivity reaction.

The Mantoux skin test should be read between 48 and 72hrs after administration. The basis of reading  is the presence or absence of induration, which may be determined by inspection and by palpation. A record should also be made of formation of vesicles, bullae, lymphangitis, ulceration and necrosis at the test site. The formation of vesicles, bullae or necrosis at the test site indicates positive result. A negative mantoux result usually signifies that the individual has never been exposed to Mycobacterium tuberculosis i.e. absence of cell mediated immunity to tuberculin.

89. The permeability of the irritable cell membrane has been increased for potassium ions during an experiment. What changes of membrane electric status can occur?

Explanation

Hyperpolarization is as a result of K+ efflux (K+ yield) making it take a longer time for the cell to reach the threshold potential. This delay in reaching the threshold produces the delay or decrease in heart rate.

Vagus nerve branches that innervate the heart produce parasympathetic effects on heart’s function e.g. ↓heart rate, force of contraction etc. This is as a result of hyperpolarization of the pacemaker cells.
90. A patient with II stage hypertension has been taking one of hypotensive medications for the purpose of treatment. After a time arterial pressure decreased, but the patient started complaining of flaccidity, sleepiness, indifference. A bit later he felt stomach pain. He was diagnosed with ulcer. What hypotensive medication has the patient been taking?

Explanation

Reserpine, a plant alkaloid is a sympatholytic; it blocks the Mg2+/adenosine triphosphate-dependent transport of biogenic amines, norepinephrine, dopamine and serotonin from the cytoplasm into storage vesicles in the adrenergic nerves of all body tissues. This causes the ultimate depletion of biogenic amines. Sympathetic function in general is impaired because of decreased release of norepinephrine (↓Arterial pressure). But studies have shown that an appropriate dose of reserpine provokes an increase in output of HCl by the stomach, leading hyperacidity in stomach which is ulcerogenic. Reports of peptic ulceration in individuals receiving reserpine and of increased gastric secretion upon administration of this drug, have engendered the suspicion that reserpine is a significant ulcerogenic agent.

91. 6 months after delivery a woman had uterine bleeding. Gynecological examination revealed in the uterine cavity a dark-red tissue with multiple cavities that resembled of \\\\\\\"sponge\\\\\\\". Microscopic examination of the tumour revealed some atypic light epithelial Langhans cells and giant cells of syncytiotrophoblast in blood lacunas. What tumour is it?

Explanation

Chorioepithelioma or choriocarcinoma or gestational choriocarcinoma is a malignant neoplasm of trophoblastic cells derived from a previously normal or abnormal pregnancy. NB: trophoblast differentiates into 2 layers (cytotrophoblast and syncytiotrophoblast) in the 2nd week of development of an embryo. Chorioepithelioma is a soft, fleshy, yellow-white tumor that usually has large pale areas of necrosis and extensive hemorrhages. Histologically, it does not produce chorionic villi and consists entirely of proliferating syncytiotrophoblasts and cytotrophoblasts.

92. A 40 y.o. patient complains of intensive heartbeats, sweating, nausea, vision impairment, arm tremor, hypertension. From his anamnesis: 2 years ago he was diagnosed with pheochromocytoma. Hyperproduction of what hormones causes the given pathology?

Explanation

Pheochromocytoma is the most common tumor of the adrenal medulla in adults. Most tumors secrete epinephrine, norepinephrine and dopamine which can cause episodic hypertension. There is increased catecholamines and metanephrines in urine and plasma. Epinephrine stimulates glycogen phosphorylase (rate-limiting enzyme in glycogenolysis i.e. breakdown of glycogen to glucose). This results in hyperglycemia (increase glucose conc. in blood). There is also pronounced sympathetic effects throughout the body like dry mouth.

93. A 56 y.o. patient has been suffering from thyreotoxicosis for a long time. What type of hypoxia can be developed?

Explanation

A peculiar variant of histotoxic (tissue) hypoxia arises during acute dissociation or uncoupling of processes of oxidation and phosphorylation reactions in respiratory chain. The agents that dissociate process of oxidation and phosphorylation are wide range of substances of exogenous and endogenous origin which includes: hormones of thyroid gland (which are excessively increased in  thyreotoxicosis), excess calcium, toxins etc. In a healthy organism, thyroid hormones (thyroxine and triiodothyronine) carry out the function of physiologic regulation of the association of oxidation and phosphorylation degree, together with other functions.

94. A sportsman was recommended to take a medication that contains carnitine in order to improve his results. What process is activated by carnitine the most?

Explanation

The major pathway for catabolism of saturated fatty acids is a mitochondrial pathway called β-oxidation. After a long-chain fatty acid (LCFA) enters a cell, it is converted in the cytosol to its Co-A derivative. Because β-oxidation occurs in the mitochondrial matrix, the fatty acid must be transported across inner mitochondrial membrane which is impermeable to Co-A. therefore, a specialized carrier transports the long chain acyl group from the cytosol into the mitochondrial matrix. This carrier is carnitine and this rate-limiting transport process is called the carnitine shuttle. Since carnitine helps the mitochondria utilize energy, it plays a critical role in reducing the occurrence and impact of obesity. In addition to helping the mitochondria burn fat as energy, carnitine is also vital for removing waste products from mitochondria. Obesity and aging contribute to low carnitine levels, which compromises mitochondrial performance and increases insulin resistance, promoting further obesity and carnitine reduction.

95. A patient has symptoms of inflammation of urogenital tracts. Examination of a vaginal smear revealed big monocellular, pear-shaped organisms with the pointed spike at the posterior end of body, big nucleus and undulating membrane. What protozoa were found in the smear?

Explanation

In the urogenital tract, the flagellate protozoa, Trichomonas vaginalis is the important pathogen. Trichomonas vaginalis causes trichomoniasis. It is a pear-shaped organism with a central nucleus and four anterior flagella. It has an undulating membrane that extends about two-thirds of its length. It exists only as a trophozoite; there is no cyst form. The organism is transmitted by sexual contact and the primary locations of the organism are the vagina and the prostate. In women, a watery, foul-smelling, greenish vaginal discharge accompanied by itching and burning occurs.

96. A patient has extrasystole. ECG shows no P wave, QRS complex is deformed, there is a full compensatory pause. What extrasystoles are these?

Explanation

        Nodal or atrioventricular (AV) extrasystole is caused by stimulus arising from AV node. The excitation of the atria differs from normal, the impulse is transmitted retrogradely from bottom to top. The following signs are characteristic of the ECG in nodal extrasystole:

·        Premature appearance of the cardiac complex.

·        Changes in the P wave which becomes negative to show the retrograde atrial excitation (in some cases the P wave is absent on the ECG).

·        If the atria and ventricles are excited synchronously, the P wave superimposes (overlaps) the QRS complex.

97. A patient who suffers from cancer of back of tongue has an intense bleeding as a result of affection of dorsal lingual artery by the tumour. What vessel should be ligated to stop bleeding?

Explanation

The lingual artery arises at the level of posterior horn of the hyoid bone from the external carotid artery. The artery occupies the lingual triangle (of Pirogov) immediately below the hyoglossus muscle. It gives the dorsal lingual branches, deep lingual artery and sublingual artery.

98. In course of an experiment a big number of stem cells of red bone marrow was in some way destructed. Regeneration of which cell populations in the loose connective tissue will be inhibited?

Explanation

Monocytes are the precursors of the cells of the mononuclear phagocytic system. Monocytes are the largest of the white blood cells in a blood smear. They travel from the bone marrow to the body tissues, where they differentiate into the various phagocytes of the mononuclear phagocytic system i.e. connective tissue macrophages (histiocytes), osteoclasts, alveolar macrophages, perisinusoidal macrophages in the liver (kupffer cells) and macrophages of lymph nodes, spleen and bone marrow, among others. Monocytes remain in the blood for only about 3 days. Monocytes transform into macrophages, which function as antigen presenting cells in the immune system.

99. Histological examination of a 40 y.o. man’s thymus revealed decreased share of parenchymatous gland elements, increased share of adipose and loose connective tissue, its enrichment with thymus bodies. The organ’s mass was unchanged. What phenomenon is it?

Explanation

There is a certain correlation between age of individual and thymus activity. In neonates, it already appears to have a considerable mass of 13.3g on the average. Most intensive growth occurs during first 3years of life, when the gland doubles in size. Having reached the maximum weight (about 26 – 30g), the thymus retains it until 20years of life. After 20 years of age, the thymus parenchyma experiences gradual involution and is substituted with fat tissue. After 50 years of age, the fat constitutes 90% of gland mass.

100. Mucous membrane of the right palatine tonsil has a painless ulcer with smooth lacquer fundus and regular cartilagenous edges. Microscopically: inflammatory infiltration that consists of lymphocytes, plasmocytes, a small number of neutrophils and epithelioid cells; endovasculitis and perivasculitis. What disease is it?

Explanation

Spirochetes are thin-walled, flexible, spiral rods. Three (3) genera of spirochetes cause human infection: Treponema (syphilis and the nonvenereal treponematoses); Borrelia (lyme disease and relapsing fever); Leptospira (leptospirosis). Treponema pallidum are thin, tight, spirals seen by darkfield illumination, silver impregnation or immunofluorescent stain.

Syphilis (lues) is a sexually transmitted disease of mankind caused by the spirochete – Treponema pallidum. Stages of syphilis are: primary (chancre); secondary (disseminated) and Tertiary (with lesions of deep organs following a latent period of 2 – 20 years or more).

The chancre develops at the site of inoculation in 10 – 90 days (average 21 days) and has a characteristic “luetic vasculitis” (endovasculitis, perivasculitis) in which endothelial cells proliferate and swell and the walls of the vessels become thickened by lymphocytes and fibrous tissue. Syphilis can also produce fibrinoid or caseous necrosis (gumma).

101. Bacteriological laboratory examines canned meat whether it contains botulinum toxin. For this purpose an extract of test specimen and antitoxic antibotulinic serum of A, B, E types were introducted to a group of mice under examination; a control group of mice got the extract without antibotulinic serum. What serological reaction was applied?

Explanation

102. To prevent postoperative bleeding a 6 y.o. child was administered vicasol that is a synthetic analogue of vitamin K. Name post-translational changes of blood coagulation factors that will be activated by vicasol:

Explanation

Vitamin K is essential for the formation of various clotting factors in the liver, namely clotting factor II, VII, IX and X; Protein C and S.

Factor II – Prothrombin

Factor VII – Stable factor

Factor IX – Christmas factor

Factor X – Stuart-Prower factor

These factors undergo vitamin K-dependent post-translational modification, whereby a number of their glutamic acid residues are carboxylated to form ɣ-carboxyglutamic acid residues. The ɣ-carboxyglutamyl residues bind calcium ions which are essential for interaction between the coagulation factors and platelet membranes. This oral anticoagulants block epoxide reductase and creation of active form of vitamin K resulting in disturbances in prothrombin and proconvertin synthesis in liver.

103. As a result of spinal-cord trauma a 33 y.o. man has a disturbed pain and temperature sensitivity that is caused by damage of the following tract:

Explanation

The spinothalamic tract is divided into the anterior spinothalamic tract in the anterior funiculus of spinal cord and lateral spinothalamic tract in the lateral funiculus of spinal cord. The anterior spinothalamic tract arises from the cells of the gelatinous substance of the apex of posterior horn and ascends to reach the thalamus. It transmits the impulses of tactile sensitivity (crude touch – protopathic sensation). The lateral spinothalamic tract arises from the cells located within the head of posterior horn and ascends to the thalamus; it transmits the impulses of pain and temperature sensitivity. Therefore, lesion of this tract leads to total loss of pain and temperature sensivity on both sides below the level of lesion.

104. For the preparation of a patient’s burn skin surface a certain medication was used. Its antiseptic action is provided by free oxygen that segregates in presence of organic substances. Choose the right answer:

Explanation

Potassium permanganate is a non-organic antiseptic and disinfectant. Its an oxidizing agent liberating oxygen, which oxidizes bacterial protoplasm. KMnO4 → MnO2 + O2.

MnO – astringent action; O2- oxidizing proteins in microbes, deodorizing action, antidote action.

Potassium permanganate is used in disinfections of impaired skin and mucous areas, irrigation of pus wounds.

·        Irrigation of purulent wounds (0.01 – 0.5% solution)

·        Gargling and mouthwash in diseases of throat and oral cavity (0.01 – 0.1% solution)

·        Syringing in gynecology and urology (0.01 – 0.1% solution)

·        Processing of burns (2 – 5% solution)

·        Lavage of stomach in acute poisoning with morphine, alcohol (0.1% solution)

Furacillin: organic antiseptic and disinfectant → furan derivative

Chlorhexidine bigluconate: nonorganic antiseptic and disinfectant → halogen → chlorine containing

Sodium hydrocarbonate: drug for transfusion therapy → drug for correction of acidosis.

105. After a 2 y.o. child has had flu, there appeared complaints about ear ache. A doctor revealed hearing impairment and inflammation of the middle ear. How did the infection penetrate into the middle ear?

Explanation

The middle ear comprises the tympanic cavity with the auditory ossicles and the pharyngotympanic (auditory or eustachian) tube. The auditory tube communicates the tympanic cavity with the nasopharynx. The tube runs inferiorly and medially; it is 3.5cm long and 2mm wide. The tube features the bony and cartilaginous parts. The tube has two openings called the tympanic opening and the pharyngeal opening. Infectious agents can get into the middle ear from the pharynx through this tube and cause complications like otitis media.

106. A 50 y.o. patient with chronic cardiac insufficiency and tachyarrhythmia was prescribed a cardiotonic drug. What drug was prescribed?

Explanation

The symptoms describe cardiac insufficiency. Cardiac glycoside is the most appropriate in this case. Cardiac glycosides are glycosidic drugs of plant origin having cardiac inotropic property, increase myocardial contractility and output. Corglyconum is short acting for acute cardiac insufficiency and administered intravenously.

Digitoxin is long-acting but basically administered orally and used mainly for chronic heart failure (also digoxin).

 

107. Microscopical examination of a removed appendix revealed an edema, diffuse neutrophilic infiltration of appendix wall along with necrosis and defect of mucous membrane with affection of its muscle plate. What appendicitis form was developed?

Explanation

Defect of mucous membrane – ulcer

Phlegmonous – ulcerative appendicitis (ulcerophlegmonous appendicitis) is characterized by phlegmonous inflammation with necrosis and ulceration in mucosa.

108. A 39 y.o. woman went through an operation in course of which surgeons removed her uterine tube that was enlarged and a part of an ovary with a big cyst. Histological examination of a tube wall revealed decidual cells, chorion villi. What was the most probable diagnosis made after examination of the uterine tube?

Explanation

Occasionally, implantation takes place outside the uterus, resulting in extrauterine pregnancy or ectopic pregnancy. Ectopic pregnancies may occur at any place in the abdominal cavity, ovary, or uterine tube (tubal pregnancy). 95% of ectopic pregnancies occur in the uterine tube (tubal pregnancy). In all abnormal locations, the fertilized ovum undergoes its usual development with the formation of placental tissue (chorionic villi), amniotic sac and fetus and the host implantation site develops decidual changes and syncytiotrophoblasts. The wall of the fallopian/uterine tube is thin and unless the ectopic pregnancy is discovered, the wall usually ruptures by the 12th week of gestation.

109. A 4 y.o. boy has had recently serious viral hepatitis. Now there are such clinical presentations as vomiting, loss of consciousness, convulsions. Blood analysis revealed hyperammoniemia. Disturbance of which biochemical process caused such pathological condition of the patient?

Explanation

          Hyperammonemia can be acquired (e.g. liver disease) or hereditary (e.g. urea cycle enzyme deficiencies). Acquired hyperammonemia may be a result of an acute process e.g. viral hepatitis, ischemia or hepatotoxins. As a result, the detoxification of ammonia (i.e. its conversion to urea) is severely impaired, leading to elevated levels of circulating ammonia.

110. Examination of a patient revealed hyperkaliemia and hyponatremia. Low secretion of which hormone may cause such changes?

Explanation

Aldosterone produced in adrenal cortex (zona glomerulosa): causes increased sodium (Na+) reabsorption; increased potassium and hydrogen ions (↑K+, H+) excretion. They increase sodium (↑Na+) channel and Na+ /K+-pump insertion in principal cells of collecting duct; enhances K+ and H+ excretion by way of principal cell K+ channels and α-intercalated cell H+ ATPases of collecting duct. Therefore, increase in aldosterone → ↑ K+ in urine (excretion) and ↓ Na+ in urine (↑ reabsorption); And decreased aldosterone → ↓ K+ excretion (↓K+ in urine) and ↓ Na+  reabsorption (i.e. ↑Na+ in urine); same effects on sweats glands too.

111. In course of indirect histogenesis of tubular bone tissue a plate is formed between epiphyseal and diaphyseal ossification centres that provides further lengthwise growth of bones. What structure is it?

Explanation

Metaphysis is a portion of the bone between epiphysis and diaphysis. During bone growth, the metaphyseal plate is responsible for the longitudinal growth of the bone. Actual lengthening of the bone occurs when new cartilage matrix is produced at this plate. Production of new cartilage matrix pushes the epiphysis away from the diaphysis, elongating the bone. The events that follow this incremental growth, namely: hypertrophy, calcification, resorption and ossification, simply involve the mechanism by which the newly formed cartilage is replaced by bone tissue during development.

112. A 40 y.o. woman was admitted to the infectious diseases department with high body temperature. Objectively: evident meningeal symptoms. A spinal cord punction was made. What anatomic formation was punctured?

Explanation

Ligamentum flava resides between the vertebral arches and consist of yellow elastic tissue. The clefts between the vertebral arches are covered by the ligamentum flava, which is the widest in the lumbar region. Therefore, these regions are used for the punctures of the vertebral canal to access the subarachnoid space. This procedure is actually performed on the L2 and L3 as well between the L3 and L4. Moreover, the puncture is also performed between the occipital bone and the first cervical vertebra piercing the atlanto-occipital membrane. In the thoracic region, the spinous processes overlap each other like a tile covering the arches of the lower vertebrae.

113. After resection of the middle third of femoral artery obliterated by a thrombi the lower extremity is supplied with blood due to the surgical bypass. Name an artery that plays the main role in reestablishment of blood flow:

Explanation

The femoral artery arises directly from the external iliac artery. The boundary between the arteries is the inferior border of the inguinal ligament. The greatest branch of the femoral artery is the deep artery of thigh. The deep femoral artery (deep artery of thigh) is nearly as wide as the femoral artery. It arises 4-5cm below the inguinal ligament. It is the main artery which supplies the thigh. Its branches are medial circumflex, lateral circumflex femoral arteries; and perforating arteries. Clinical applications: the deep artery of thigh is of great importance for collateral circulation. Intrinsic anastomoses and intersystem anastomoses (with the internal iliac and popliteal arteries) provide good conditions for collateral circulation, which is vital in occlusions of femoral artery.

114. A patient’s knee joint doesn’t extend, there is no knee-jerk reflex, skin sensitivity of the anterior femoral surface is disturbed. What nerve structures are damaged?

Explanation

The muscles of the thigh are divided into the anterior, medial and posterior groups. The anterior group – flexors of the thigh; posterior group – extensors of the thigh and medial group – adductors. The quadriceps femoris is the strongest muscle of the thigh, which occupies the entire anterior, lateral and partially medial surfaces of the thigh. It consists of four heads: rectus femoris, vastus medialis, vastus intermedius and vastus lateralis. All four heads of the quadriceps femoris in the lower part of the thigh fuse together and continue as a common tendon. This muscle acts on the hip joint and participates in the flexion of the thigh; it’s also a powerful extensor of the leg at the knee joint. It is supplied by the femoral nerve.

115. An electron micrograph of a kidney fragment presents an afferent arteriole. Under its endothelium some big cells can be seen that contain secretory granules. What type of cells is it?

Explanation

           Juxtaglomerular apparatus is formed by 3 different structures:

·        Macula densa: is the end portion of thick ascending segment as it opens into the distal convoluted tubule. It is situated between the afferent and efferent arterioles of the same nephron. It is very close to afferent arteriole. Macula densa is formed by tightly packed cuboidal epithelial cells.

·        Mesangial cells: are situated in the triangular region bound by afferent arteriole, efferent arteriole and macula densa.

·        Juxtaglomerular cells: are specialized smooth muscle cells situated in the wall of afferent arteriole just before it enters the Bowman’s capsule.

116. For the purpose of retrospective diagnostics of recent bacterial dysentery it was decided to perform serological examination of blood serum in order to determine antibody titer towards Shiga bacilli. What of the following reactions should be applied?

Explanation

Hemagglutination tests: many viruses clump red blood cells from one species or another (active hemagglutination). This can be inhibited by antibody specifically directed against the virus (hemagglutination inhibition) and can be used to measure the titer of such antibody. Red blood cells (RBCs) also can absorb many antigens and when mixed with matching antibodies, they will clump (this is known as passive hemagglutination, because the Red cells are passive carriers of the antigen).

117. Examination of a 43 y.o. anephric patient revealed anemia symptoms. What is the cause of these symptoms?

Explanation

Erythropoietin is the most important general factor for erythropoiesis. It is also called hemopoietin or erythrocyte stimulating factor. Major quantity of erythropoietin is secreted by peritubular capillaries in kidney. Erythropoietin causes formation and release of new RBCs into circulation. It promotes production of proerythroblasts from CFU-E of bone marrow; development of proerythroblasts into matured RBCs and release of matured erythrocytes into blood. Therefore, anephric (absence of kidney) patient → ↓erythropoietin → ↓RBC → Anemia.

118. A 50 y.o. patient was admitted to the hospital with complaints about pain behind his breastbone, asphyxia during physical activity. Angiography revealed pathological changes in the posterior interventricular branch of the right coronary artery. What heart parts are affected?

Explanation

The greatest branch of the right coronary artery is the posterior interventricular branch which runs along the sulcus of the same name (posterior interventricular sulcus) and anastomoses with the anterior interventricular branch of the left coronary artery. It supplies the entire posterior wall of the right ventricle, a part of the posterior wall of left ventricle and posterior one third of the interventricular septum. A pathological change in this artery will affect the part of the heart it supplies.

119. 48 hours after performing tuberculin test (Mantoux test) to a child a 10 mm papule appeared on the spot of tuberculin introduction. What hypersensitivity mechanism underlies these changes?

Explanation

Mantoux test is a type IV Hypersensitivity reaction (HSR), which involves macrophages,T-lymphocytes and lymphokines(cytokines). Mononuclear cells (lymphocytes,monocytes,macrophages).

The Mantoux skin test should be read between 48 and 72hrs after administration. The basis of reading  is the presence or absence of induration, which may be determined by inspection and by palpation. A record should also be made of formation of vesicles, bullae, lymphangitis, ulceration and necrosis at the test site. The formation of vesicles, bullae or necrosis at the test site indicates positive result. A negative mantoux result usually signifies that the individual has never been exposed to Mycobacterium tuberculosis i.e. absence of cell mediated immunity to tuberculin.

120. Examination of a newborn boy’s genitals revealed a cleft of urethra that opens on the inferior surface of his penis. What developmental anomaly is it?

Explanation

Defects in male genitalia:

Hypospadia: fusion of the urethral folds is incomplete and abnormal openings of the urethra occur along the inferior (undersite) surface of the penis, usually near the glans, along the shaft or near the base of the penis.

Epispadia is a rare abnormality (1/30000 births) in which the urethral meatus is found on the dorsum (superior) surface of the penis.

121. A patient has a disturbed absorption of fat hydrolysates. It might have been caused by a deficit in the small intestine cavity:

Explanation

Steatorrhea is the formation of bulky, foulsmelling, frothy and clay-coloured stools with large quantity of undigested fat because of impaired digestion and absorption of fat in the intestine. Any condition that causes indigestion or malabsorption of fat leads to steatorrhea. Causes of steatorrhea are: lack of pancreatic lipase, liver disease affecting the secretion of bile (lack of bile acid or obstruction of bile duct), celiac disease and cystic fibrosis.
122. Inhabitants of territories with cold climate have high content of an adaptive thermoregulatory hormone. What hormone is meant?

Explanation

Thyroid hormones (thyroxine) increase the metabolic activities in most of the body tissues except brain, retina, spleen, testes and lungs. It increases basal metabolic rate (BMR) by increasing the O2 consumption of the tissues. In connection with this, thyroid hormone increases the heat production in the body, by accelerating various cellular metabolic processes and increasing BMR. It is called thyroid hormone-induced thermogenesis. During hypersecretion of thyroxine, the body temperature increases greatly. This is an adaptive hormone which is increased in inhabitants of cold climate.

123. Violation of safety rules resulted in calomel intoxication. Two days later the daily diuresis was 620 ml. A patient experienced headache, vomiting, convulsions, dyspnea, moist rales in lungs. What pathology is it?

Explanation

124. A newborn child with pylorostenosis has often repeating vomiting accompanied by apathy, weakness, hypertonicity, sometimes convulsions. What disorder form of acid-base balance is it?

Explanation

Acidosis is the reduction in pH (increase in H+ concentration) below normal range. pH is less than 7.35; it is produced by:

·        Increase in partial pressure of CO2 in the body.

·        Decrease in HCO3- concentration.

Alkalosis is the increase in pH (decrease in H+ concentration) above normal range. pH is greater than 7.45; it is produced by:

·        Decrease in partial pressure of CO2 in the body.

·        Increase in HCO3- concentration.

Each of these two disorders has respiratory and non-respiratory forms. The non-respiratory form is divided into metabolic and excretory(non-gaseous).

·        Respiratory acidosis is the acidosis that is caused by alveolar hypoventilation e.g. airway obstruction due to bronchitis or lung diseases (pneumonia).

·        Respiratory alkalosis is caused by alveolar hyperventilation e.g. hypoxia in high altitude.

·        Non-respiratory:

-Metabolic acidosis is characterized by excess accumulation of organic acids such as lactic acid, ketoacids and uric acid formed by normal metabolism e.g. in Diabetes mellitus or extreme/prolonged exercise.

-Excretory/Non-gaseous acidosis may develop in impaired renal H+ excretion related to increased loss of bicarbonate in urine; diarrhea causes acidosis by the loss of bicarbonate with faeces.

-Excretory/Non-gaseous alkalosis: vomiting (loss of gastric acid), increased metabolism of lactate and citrate (turns into bicarbonate and water), long-term use of thiazides and loop diuretics.

      It is excretory/non-gaseous alkalosis because of the frequent vomiting.

125. Glutamate decarboxylation results in formation of inhibitory transmitter in CNS. Name it:

Explanation

       Certain amino acids undergo decarboxylation that means the removal of their α-carboxyl group resulting in liberation of CO2 and formation of biogenic amines. Biogenic amines are physiologically active substances such as hormones, neurotransmitters etc. decarboxylation of amino acids:

·        Tryptophan → Niacin → NAD+/NADP+

          Tryptophan → Serotonin →Melatonin

·        Histidine → Histamine

·        Glutamine → GABA

         Glutamine → Glutathione

126. In course of histidine catabolism a biogenic amine is formed that has powerful vasodilating effect. Name it:

Explanation

       Certain amino acids undergo decarboxylation that means the removal of their α-carboxyl group resulting in liberation of CO2 and formation of biogenic amines. Biogenic amines are physiologically active substances such as hormones, neurotransmitters etc. decarboxylation of amino acids:

·        Tryptophan → Niacin → NAD+/NADP+

          Tryptophan → Serotonin →Melatonin

·        Histidine → Histamine

·        Glutamine → GABA

          Glutamine → Glutathione

127. Utilization of arachidonic acid via cyclooxigenase pathway results in formation of some bioactive substances. Name them:

Explanation

Cycloxygenase, the enzyme which converts arachidonic acid into the endoperoxide precursors of prostaglandin, has at least two different  isoforms: COX-1 & COX-2. COX-1 is primarily expressed in non-inflammatory cells whereas COX-2 is expressed in activated lymphocytes, polymorphonuclear cells and other inflammatory cells. Acetylsalicyclic acid and the older non-selective NSAIDs inhibit both cyclooxygenase isoforms & thereby decrease prostaglandin & thromboxane synthesis. 
128. As a result of damage to certain structures of brainstem an animal lost orientation reflexes. What structures were damaged?

Explanation

       Quadritubercular bodies/tectal plate is the dorsal portion of the midbrain comprises 4 colliculi – two superior and two inferior. The nuclei of colliculi are responsible for reflexes associated with sudden sound and visual stimuli (auditory and visual orientative reflexes); they also maintain consciousness. The nuclei give rise to the tectospinal tract. Superior colliculi – visual orientative reflex; Inferior colliculi – auditory orientative reflex.

129. Osmotic pressure of a man’s blood plasma is 350 mosmole/l (standard pressure is 300 mosmole/l). First of all it will result in high secretion of the following hormone:

Explanation

       Antidiuretic hormone (vasopressin) is secreted in response to decrease blood volume and increase plasma osmolarity. It binds to receptors on principal cells of collecting ductules causing increase number of aquaporins and increase water reabsorption which leads to decreased diuresis.

130. A young man consulted a doctor about disturbed urination. Examination of his external genitals revealed that urethra is split on top and urine runs out of this opening. What anomaly of external genitals development is the case?

Explanation

Defects in male genitalia:

Hypospadia: fusion of the urethral folds is incomplete and abnormal openings of the urethra occur along the inferior (undersite) surface of the penis, usually near the glans, along the shaft or near the base of the penis.

Epispadia is a rare abnormality (1/30000 births) in which the urethral meatus is found on the dorsum (superior) surface of the penis.

131. A patient complains of pain in the area of his liver. Duodenal intubation revealed yellowish, oval, narrowed at the poles eggs with an operculum at the end. Size of these eggs is the smallest among all helminth eggs. What is the most probable diagnosis?

Explanation

Opisthorchosis is a trematode (fluke) infection caused by infection with one of the species of the liver fluke Opisthorchis, which is acquired by eating raw or undercooked freshwater fish containing infectious metacercariae. Opisthorchis sinensis is still widely known as Clonorchis sinensis. Adult flukes lives in biliary ducts. Intermediate hosts – snail and fish. Diagnostic features of eggs – operculated. Endemic area – Asia.

132. Colonoscopy of a patient ill with dysentery revealed that mucous membrane of his large intestine is hyperemic, edematic, its surface was covered with grey-and-green coats. Name the morphological form of dysenteric collitis:

Explanation

Morphology of dysenteric colitis has 4 stages: catarrhal colitis; fibrinous colitis; ulcer formation (ulcerative colitis); healing of the wound. During the stage of fibrinous colitis, within the course of 24hours, a fibrinosuppurative exudate first patchily, then diffusely covers the mucosa and produces a dirty grayish pseudomembrane, consisting of necrotic mucosa, neutrophils, fibrin and erythrocytes. Sloughed pseudomembrane, together with blood-tinged mucus, comprises the characteristic dysenteric stool.

133. 24 hours after appendectomy blood of a patient presents neutrophilic leukocytosis with regenerative shift. What is the most probable mechanism of leukocytosis development?

Explanation

Neutrophilic leukocytosis: increase in neutrophil count. Depending on the proportion between the mature and premature forms of neutrophils, two types of the nuclear shift may be distinguished: to the left, when there is an elevated content of immature forms of neutrophilic granulocytes (myelocytes, metamyelocytes, bands) in the blood; And to the right, when the mature neutrophils with a large number of segments (5 – 6) prevail against a background of younger cells disappearing. The nuclear shift may be subdivided into: regenerative, hyperregenerative, degenerative and regenerative-degenerative shifts.

Regenerative shift develops against a background of mild general leukocytosis, accompanied with elevated content of bands and metamyelocytes (immature forms). This shift results from the reactive leukopoiesis (granulocytopoiesis) activation i.e. increased synthesis or amplification of leukopoiesis and is common in suppurative septic processes like appendicitis.

134. Arterial pressure of a surgeon who performed a long operation rised up to 140/110 mm Hg. What changes of humoral regulation could have caused the rise of arterial pressure in this case?

Explanation

     Stress activates the sympathoadrenal system and the hypothalamic-pituitary-adrenocortical (HPA) axis. The stress factor here is the surgery. Defense reactions involve catecholamine release, vagal withdrawal, cortisol secretion and activation of the rennin-angiotensin system. Catecholamine release is capable of increasing the arterial pressure. Sympathoadrenal system involves the sympathetic nervous system and the adrenal glands especially increased sympathetic activity that causes increased secretion of epinephrine by the adrenal medulla and norepinephrine by the postganglionic sympathetic nerve endings.

135. A hypertensive glucose solution was introduced to a patient. It will intensify water movement:

Explanation

Albumin maintains the oncotic pressure in the blood vessels, it does not go out of the blood vessels. Injection of concentrated solution of albumin will increase the oncotic pressure in the blood vessel thereby causing water to move from the interstitial/intercellular space to the blood vessel (i.e. the capillaries).

    In contrast, glucose can pass through the capillaries into the interstitial/intercellular space but it does not readily enter the cell. Therefore, it increases the oncotic pressure in the interstitial space and cause water to move from the intracellular (inside the cell) space to the interstitial/intercellular space.

136. A 36 y.o. man has a craniocerebral trauma. Objectively: diminished breath sounds, thready pulse, no reflexes. What way of pyracetam introduction will be the most appropriate in this case?

Explanation

Pyracetam (Nootropil) is a nootropic. Nootropic drugs are drugs which improve cerebral blood flow and optimize energetic metabolism in neurons, increase brain stability in hypoxia, stimulate higher cortical function, memory and education processes. It is administered 20% - 5ml of solution. Craniocerebral trauma is an emergency case; so for fast onset of action, it is administered intravenously. Although, it can also be administered orally or intramuscularly.

137. A child complains of general weakness, loss of appetite, a troubled sleep, itching in the perianal area. The provisional diagnosis is enterobiasis. In order to specify this diagnosis it is necessary to perform:

Explanation

 Enterobius vermicularis causes pinworm infection (enterobiasis). The life cycle is confined to humans. The infection is acquired by ingesting the worm eggs. Perianal pruritus is the most prominent symptom. Pruritus is thought to be an allergic reaction to the presence of either the adult female or the eggs. Scratching predisposes to secondary bacterial infection. In Laboratory diagnosis, the eggs are recovered from perianal skin by using the scotch tape technique and can be observed microscopically. Unlike those of other intestinal nematodes, these eggs are not found in the stools. There are no means of prevention. It is treated with mebendazole.

138. A patient was ill with burn disease that was complicated by DIC syndrome. What stage of DIC syndrome can be suspected if it is known that the patient’s blood coagulates in less than 3 minutes?

Explanation

Disseminated intravascular coagulation (DIC) is apathological syndrome characterized by formation of disseminated blood clots in the microcirculatory bed (often in combination with simultaneous reduction of blood coagulability) causing hemorrhages. Stages includes:

·        At the first stage (hypercoagulation): it is characterized by generalized increase of blood coagulation in the microvessels (hypercoagulation). Large number of fibrin clots are formed. They close the vessel (fibrinoembolism).

·        At the second stage (Transitive): the amount of thrombocytes, fibrinogen, prothrombin in the blood decreases sharply because they have already been used at the first stage with the resultant consumption coagulopathy. Thus, hemorrhagic syndrome develops.

·        At the third stage (hypocoagulation): fibrinolysis activation takes place in response to generalized increase of coagulation occurring at the first stage which makes hemorrhagic syndrome more severe.

139. In course of practical training students studied a stained blood smear of a mouse with bacteria phagocyted by leukocytes. What cell organella completes digestion of these bacteria?

Explanation

Lysosomes are small organelles containing digestive enzymes, their derivatives include phagosomes, phagolysosomes, autophagosomes and autophagolysosomes. Lysosomes are digestive organelles rich in hydrolytic enzymes such as proteases, nucleases, glucosidases, lipases and phospholipases. 3 pathways for digestion exist:

·        Extracellular large particles such as bacteria, cell debris and other foreign materials are engulfed in the process of phagocytosis. A phagosome, formed as the material is internalized within the cytoplasm, subsequently fuses with a lysosome to create a phagolysosome.

·        Extracellular small particles such as extracellular proteins, plasma membrane proteins and ligand-receptor complexes are internalized by endocytosis and receptor mediated endocytosis.

·        Intracellular particles such as entire organelles, cytoplasmic proteins and other cellular components – autophagy.

140. A 55 y.o. woman consulted a doctor about having continuous cyclic uterine hemorrhages for a year, weakness, dizziness. Examination revealed skin pallor. Hemogram: Hb- 70 g/l, erythrocytes - 3, 2 · 1012/l, color index - 0,6, leukocytes - 6, 0 · 109/l, reticulocytes - 1%; erythrocyte hypochromia. What anemia is it?

Explanation

Chronic post-hemorrhagic anemia frequently develops after long repeated slow blood loss, in the majority of cases at hemorrhages from GIT (ulcer, cancer, hemorrhoids), uterine bleedings, in hemophilia, hemorrhagic diathesis etc. A clinic-morphological sign of anemia is a pale skin (skin pallor) and visceral organs. Severe iron-deficiency may follow.

141. According to audiometry data a patient has a disturbed perception of medium-frequency sounds. It might have been caused by a damage of:

Explanation

Two membranes divide the spiral canal of cochlea into 3 compartments – vestibular and basilar membrane. Along the basilar membrane are 20,000-30,000 tiny fibers called basilar fibers. Each fiber has different size and shape. Fibers near the oval window (base) are short and stiff (narrowest), while approaching towards helicotrema (apex), the basilar fibers gradually become longer and soft (widest). The motion of the basilar membrane is generally described as a travelling wave. The parameters of the membrane (stiffness, narrow, wide) at a given point along its length determine its characteristic frequency at which it is most sensitive to sound vibrations. High frequency sounds localize near the base of the cochlea (near the round and oval windows), while low frequency sounds localize near the apex (helicotrema); and the middle frequency sounds – middle part of helix.

142. A patient diagnosed with carcinoid of bowels was admitted to the hospital. Analysis revealed high production of serotonin. It is known that this substance is formed of tryptophane amino acid. What biochemical mechanism underlies this process?

Explanation

 Certain amino acids undergo decarboxylation that means the removal of their α-carboxyl group resulting in liberation of CO2 and formation of biogenic amines. Biogenic amines are physiologically active substances such as hormones, neurotransmitters etc. decarboxylation of amino acids:

·        Tryptophan → Niacin → NAD+/NADP+

          Tryptophan → Serotonin →Melatonin

·        Histidine → Histamine

·        Glutamine → GABA

         Glutamine → Glutathione
143. A 23 y.o. patient complains of weakness, temperature rise up to 38 − 40oC. Objectively: liver and spleen are enlarged. Hemogram: Hb- 100 g/l, erythrocytes - 2, 9 · 1012/l, leukocytes - 4, 4 · 109/l, thrombocytes - 48 · 109/l, segmentonuclear neutrophils - 17%, lymphocytes - 15%, blast cells - 68%. All cytochemical reactions are negative. Make a hematological conclusion:

Explanation

Presence of blast cells (immature blood cells) in the blood is a sign of a malignant process in the blood forming tissue. This is undifferentiated leukosis because it is not specified if it is a lymphoblastic or myeloblastic leukosis.

144.

A patient ill with bronchial asthma didn’t inform his doctor that he had attacks of stenocardia. Doctor administered him a medication, which taking resulted in less frequent attacks of bronchial asthma, but stenocardia attacks became more frequent. What medication was administered?

Explanation

Isadrin (Isoproterenolum) is a non-selective β12 agonists (β-adrenomimetics); While it will cause bronchodilation and relieve bronchial asthma; it will also increase heart rate and therefore exacerbate stenocardia. It produces an intense stimulation of the heart to increase its arte and force of contraction causing increased cardiac output. Therefore, it is contraindicated in stenocardia (angina pectoris), hypertension, coronary artery disease, cardiac arrhythmia etc. Stenocardia needs drugs that will reduce heart work (like β-blockers, organic nitrates, calcium channel blockers) and not drugs that will increase heart work (↑HR, ↑force of contraction).

145.

A patient who has been suffering from cardiac insufficiency for several months has been taking digoxin on an outpatient basis. At a certain stage of treatment there appeared symptoms of drug overdose. What phenomenon underlies the development of this complication?

Explanation

Accumulation is enhancing of the effects in repeated doses that leads to the appearance of toxic effects. It can be:

·        Material accumulation: is accumulation of drug in the manner of bindings with proteins, phospholipids etc.

·        Functional accumulation is accumulation of the drug’s effect in spite of its elimination.

146. A patient who has been treated with diazepam on account of neurosis complains of toothache. Doctor administered him an analgetic, but its dose was lower than average therapeutic dose. What phenomenon did the doctor take into account while prescribing the patient an underdose?

Explanation

Diazepam is a benzodiazepine. It acts by increasing GABAA action (neuroinhibitory effect). An analgetic also has a neuroinhibitory effect. Therefore, taking both drugs simultaneously will potentiate the other causing a tremendous nervous inhibition. Therefore, a particular dose is to be reduced to avoid over inhibiting the nervous system.

147.

A child’s blood presents high content of galactose, glucose concentration is low. There are such presentations as cataract, mental deficiency, adipose degeneration of liver. What disease is it?

Explanation

Classic galactossemia: absence of galactose 1-phosphate uridyltransferase. It is an autosomal  recessive disorder. It causes galactosemia and galactosuria, vomiting, diarrhea and jaundice. There is accumulation of galactose 1-phosphate and galacitol in nerve, lens, liver and kidney tissue causes liver damage, severe mental retardation and cataracts.

Also galactokinase deficiency can cause elevation of galactose in blood (galactosemia) and urine (galactosuria). This is a rare autosomal recessive disorder. It causes galacitol accumulation if galactose is present in diet.

148. According to clinical indications a patient was administered pyridoxal phosphate. What processes is this medication intended to correct?

Explanation

Vitamin B6 (pyridoxine): converted to pyridoxal phosphate (PLP), a cofactor used in transamination (e.g. in ALT and AST), decarboxylation reactions, glycogen phosphorylase. Synthesis of cystathione, heme, niacin (Vit. B3), histamine and neurotransmitters including serotonin, epinephrine, norepinephrine, dopamine and GABA. Deficiency produces convulsions, hyperirritability, peripheral neuropathy (deficiency inducible by isoniazid and oral contraceptives), sideroblastic anemia due to impaired hemoglobin synthesis and iron excess.

149. In course of an operation surgeon removed a part of a lung that was ventilated by a tertiary bronchus accompanied by branches of pulmonary artery and other vessels. What part of a lung was removed?

Explanation

         The main (primary or first-order) bronchus branches in the hilum of lungs to form the lobar bronchi (secondary), which ventilate the respective lobe. The lobar bronchi in turn branch to form the tertiary (third-order) bronchi called the segmental bronchi. They ventilate the pulmonary areas called segments. Segments are the pyramidal shaped portions of lungs with apices facing the lung root and bases at the lung surface. Each segment excluding the segmental bronchus has a segmental branch of the pulmonary artery respective to segmental bronchi branches. The branches of pulmonary veins run between the segments. Ant segment is surgically removable without major damage to neighboring segments.

150. A patient with clinical presentations of immunodeficiency went through immunological examinations. They revealed significant loss of cells that form rosettes with erythrocytes of a ram. What conclusion can be made according to the analysis data?

Explanation

Erythrocyte (RBC) rosetting is a phenomenon seen through a microscope where RBCs are arranged around a central cell to form a cluster that looks like a flower. This formation occurs due to an immunological reaction between an epitope on the central cells surface and a receptor or antibody on a red blood cell. The presence of erythrocyte resetting can be used as a test for T cells. It is used in the identification of T cells where a T cell CD2 surface protein is bound to a sugar based CD58 homologue on the surface of a sheep RBC. Because the CD58 homologue is only present on the surface of sheep RBCs, other species’ RBCs cannot be used in this type of resetting. Therefore, a decrease in the number of cells that form rosettes with sheep’s RBCs indicates a decrease in T-lymphocyte level.

151. A woman with (B), Rh blood group born a child with (A) blood group. The child is diagnosed with hemolytic disease of newborn as a result of rhesus incompatibility. What blood group is the child’s father likely to have?

Explanation

152. A 45 y.o. woman suffers from Cushing’s syndrome - steroid diabetes. Biochemical examination revealed: hyperglycemia, hypochloremia. Which of the under-mentioned processes is the first to be activated?

Explanation

Cushing\'s syndrome is characterized by increased glucocorticoids. Glucocorticoids (prednisolone) stimulate gluconeogenesis. As a result, blood sugar rises, muscle protein is catabolized and insulin secretion is stimulated. Corticosteroids cause increased gluconeogenesis, increased lipolysis, CNS effects - at times including euphoria, maintenance of cardiovascular function by potentiation of norepinephrine. Gluconeogenesis uses the reversible reactions from glycolysis and 4 distinct reactions that circumvent the ones from glycolysis that are irreversible. These reactions are catalyzed by pyruvate carboxylase, phosphoenolpyruvate carboxykinase, fructose-1, 6-bisphosphatase and glucose-6-phosphatase. Glucagon, epinephrine and glucocorticoids stimulate all these enzymes (excluding pyruvate carboxylase).

153.

Histological specimen presents a vessel the wall of which consists of endothelium, basal membrane and loose connective tissue. What type of vessel is it?

Explanation

The tunics of veins are not as distinct or well defined as the tunics of arteries. Veins are divided into 3 types:

* Small veins/venules: postcapillary and muscular venules

* Medium veins

* Large veins

Arteries:

·        Large or elastic arteries

·        Medium or muscular arteries

·        Small arteries and arterioles

Muscular venules are distinguished from postcapillary venules by the presence of a tunica media. Postcapillary venules possess an endothelial lining with its basal lamina and pericytes. Postcapillary venules have no true tunica media.

·        Tunica intima: consists of endothelium with its basal lamina

·        Tunica media: smooth muscle cells

·        Tunica adventitia: collagen fibers

A vessel without tunica media, also lack muscular tissue. Muscular artery, arteriole and artery of mixed type all have tunica media. Only capillaries and postcapillary venules lack tunica media.

Also a prominent internal elastic membrane helps to distinguish muscular arteries from elastic arteries and muscular venules.

154. Autopsy of a 48 y.o. man revealed a round formation 5 cm in diameter with clear-cut outlines in the region of the 1st segment of his right lung. This formation was encircled with a thin layer of connective tissue full of white brittle masses. Make a diagnosis of the secondary tuberculosis form:

Explanation

Secondary tuberculosis usually results from reactivation of dormant, endogenous tubercle bacilli in a sensitized patient who has had previous contact with the tubercle bacillus. Reactivation typically begins in the apical or posterior segments (often 1st and 2nd segments) of one or both upper lobes (“simon’s foci”), where the organisms were seeded during the primary infection. There are 8 forms or stages of the secondary tuberculosis: Acute local tuberculosis; Fibrous local tuberculosis; Infiltrative tuberculosis; Tuberculoma; Caseous pneumonia; Acute cavernous tuberculosis; Fibrous cavernous tuberculosis; Cirrhotic tuberculosis.

Tuberculoma consists of focus necrosis surrounded by fibrous capsule. Size of tuberculoma may be near 2-5cm.

Acute cavernous tuberculosis develops due to lyses of caseous necrosis and characterized by formation of the round cavity.

Fibrous cavernous tuberculosis is the most frequent form. Macroscopically, the lesions are spherical and cavitary (cavity can contain blood and blood clots); the so-called coin lesions. Microscopically, the outer wall of cavity shows fibrosis or sclerosis. Internal surface may be connected with bronchus. The wall of cavern has 3 membranes:

·        Internal membrane occurs by necrotic tissue

·        Middle membrane occurs by special granular tissue

·        External membrane occurs by connective fibrous tissue

155.

On the 6th day of treatment a patient with acute renal insufficiency developed polyuria. Diuresis intensification at the beginning of polyuria stage of acute renal insufficiency is caused by:

Explanation

Stages of Acute renal Insufficiency: initiating stage (shock); maintenance stage (oliguric phase, 2 - 9days) and the Recovery stage (polyuric phase, 10 – 21days). The polyuric phase is ushered by a steady increase in urine volume that may reach up to 3 liters per day. Regeneration of renal tubular epithelium takes place with removal of dead material by phagocytic cells, as well as in the form of casts in urine. As tubules open up and glomerular blood flow increases, patients develop polyuria. This is because the regenerated tubular cells are undifferentiated and have not developed the specializations necessary for reabsorption of electrolytes and water.

156.

Long-term starvation cure of a patient resulted in diminished ratio of albumines and globulines in plasma. What of the following will be result of these changes?

Explanation

Normal erythrocyte sedimentation rate (ESR): Men: 2 – 10mm/h

                                                                               Women: 2 – 15mm/h

Products of inflammation (plasma proteins e.g. fibrinogen) coat RBCs and cause aggregation. The denser RBC aggregates, fall at a faster rate within a pipette tube.

↑ESR: most anemias, infections, inflammation (e.g. temporal arteritis), cancer (e.g. multiple myeloma), pregnancy, autoimmune disorders (e.g. SLE)

↓ESR: sickle cell anemia (altered shape), polycythemia (↑RBCs “dilute” aggregation factors), microcytosis, hypofibrinogenemia

157. An experimental animal was first sensibilized whereupon an antigen dose was introduced subcutaneously. This injection resulted in the development of a fibrinous inflammation with alteration of vessel walls, basal substance and fibrous structures of connective tissue in form of mucoid and fibrinoid swelling and necrosis. What immunological reaction took place?

Explanation

Type I (Immediate, Anaphylaxis, Reagin): IgE (immunoglobulin E)-dependent activation of mast cells/basophils, usually accompanied by eosinophilia e.g. urticaria (hives), hay fever, asthma (wheezing), rhinitis and conjunctivitis (stuffy nose and itchy eyes; usually seasonal)

Type II (cytotoxic): antibody dependent reactions e.g. Goodpasture syndrome, Myasthenia gravis, Graves disease, ABO hemolytic disease of newborn etc.

Type III (immune-complex): deposition of antigen-antibody complexes e.g. systemic lupus erythromatous (SLE), Arthus reaction, serum sickness, poststreptococcal glomerulonephritis etc.

Type IV (cell mediated, delayed): antibody-independent T-cell mediated reactions e.g. positive mantoux reaction (tuberculin test), hashimoto’s thyroiditis or transplant rejection etc.

158.

A patient suffers from vision impairment - hemeralopy (night blindness). What vitamin preparation should be administered the patient in order to restore his vision?

Explanation

Rhodopsin or visual purple is the photosensitive pigment of rod cells. It is made up of a protein called opsin and a chromophore. Opsin present in rhodopsin is known as scotopsin. Chromophore is a chemical substance that develops colour in the cell. Chromophore present in the rod cells is called retinal. Retinal is the aldehyde of vitamin A or retinol. Vitamin A is the name given to a group of related compounds that include retinol (vitamin A alcohol); retinal (vitamin A aldehyde) and retinoic acid (vitamin A acid). Rod cells are responsible for dim light vision or night vision or scotopic vision. Therefore, Vitamin A (retinol acetate) can treat and prevent night blindness.

159.

A 1 y.o. child with symptoms of muscle affection was admitted to the hospital. Examination revealed carnitine deficit in muscles. Biochemical base of this pathology is disturbed process of:

Explanation

The major pathway for catabolism of saturated fatty acids is a mitochondrial pathway called β-oxidation. After a long-chain fatty acid (LCFA) enters a cell, it is converted in the cytosol to its Co-A derivative. Because β-oxidation occurs in the mitochondrial matrix, the fatty acid must be transported across inner mitochondrial membrane which is impermeable to Co-A. therefore, a specialized carrier transports the long chain acyl group from the cytosol into the mitochondrial matrix. This carrier is carnitine and this rate-limiting transport process is called the carnitine shuttle. Since carnitine helps the mitochondria utilize energy, it plays a critical role in reducing the occurrence and impact of obesity. In addition to helping the mitochondria burn fat as energy, carnitine is also vital for removing waste products from mitochondria. Obesity and aging contribute to low carnitine levels, which compromises mitochondrial performance and increases insulin resistance, promoting further obesity and carnitine reduction.

160.

 A patient suffers from severe postoperative pseudomonadous infection. What of the following antibiotics should be administered in this case?

Explanation

Aminoglycosides (streptomycini sulfas – 1st generation; Gentamycini sulfas – 2nd generation; Amikacin sulfate – 3rd generation; Arbekacin sulfate - 4th generation). They are protein synthesis inhibitors (antibiotics) and bactericidal. They cause irreversible inhibition of initiation complex through binding of the 30S ribosomal subunit, can also cause misreading of mRNA and also block translocation. Therapeutic uses include: Pneumonia, chronic urinary tract infections, Tularemia and infections due to Pseudomonas aeruginosa.

161.

  A 9 m.o. child has delayed dentition, it is also out of order. Upper jaw configuration is horizontal (\"high\" palate); microscopically - irregular mineralization of tooth enamel, wrinkled enamel prisms, some of them are vacuolized. Predentin zone is extended; there are solitary denticles. What disease is it?

Explanation

Vitamin D deficiency causes a net demineralization of bone and teeth resulting in rickets in children and osteomalacia in adults. Rickets is characterized by the continued formation of the collagen matrix of bone but incomplete mineralization resulting in soft, pliable bones and teeth. It is referred to as early rickets probably because the child is 9 months old.

162.

Examination of a patient revealed an abscess of pterygopalatine fossa. Where can the infection spread to unless the disease is managed in time?

Explanation

Pterygopalatine fossa is bounded by the maxilla in the front, pterygoid process in the back and by the perpendicular plate of the palatine bone medially. The orbit is a large cavity which resembles the shape of a pyramid. The orbit communicates with the pterygopalatine fossa and the infratemporal fossa via the inferior orbital fissure.

163.

Microscopical renal examination of a 36 y.o. woman who died from renal insufficiency revealed in the glomerules proliferation of capsule nephrothelium as well as of podocytes and phagocytes accompanied by formation of \\\"crescents\\\", capillary loop necrosis, fibrinous thrombs in their lumens; sclerosis and hyalinosis of glomerules, atrophy of tubules and fibrosis of renal stroma. What is the most probable diagnosis?

Explanation

We have acute, subacute and chronic glomerulonephritis. Subacute glomerulonephritis does not necessarily follow acute attacks. It is usually characterized by the general symptoms of glomerulonephritis like proteinuria, hematuria, edema and azotemia persisting for many days and even weeks. Renal changes include those of rapidly progressive and membranoproliferative glomerulonephritis.    

164.

A forensic medical expert examines the body of a 58 y.o. man who had been consuming large amounts of alcohol for a long time and died at home. Microscopically: the right lung is dense and enlarged, its incision revealed that the tissue is greyish and homogenous, pleura is covered with greyish layers. Microscopically - alveolar cavities contain fibrin, hemolyzed erythrocytes. Make a diagnosis:

Explanation

165. Two days after consumption of smoked pork a patient got face and eyelid edemata, gastrointestinal disturbances, abrupt temperature rise, muscle pain. Blood analysis showed full-blown eosinophilia. What helminth could the patient be infected with?

Explanation

Trichinosis: causative agent – Trichinella spiralis. Transmission – fecal-oral; undercooked meat (especially pork). Findings: fever, vomiting, nausea, periorbital edema, myalgia. These findings are specific for Trichinosis.

166.

After a tooth extraction a patient felt persistent pain behind his breast bone. After sublingual intake of an antianginal drug the pain behind the breast bone disappeared, but the patient complained of headache and dizziness. What drug are these properties typical for?

Explanation

     Administered nitrates → ↑ Nitrites →↑ Nitric oxide (NO) → ↑cGMP → ↑dephosphorylation of myosin chains → vascular smooth muscle relaxation. Nitroglycerin is an organic nitrates. Nitrates inhibit coronary vasoconstriction or spasm, increasing perfusion of the myocardium and thus relieving vasospastic angina. Because of this action, nitrates are effective in treating effort-induced (stress) angina. Classic, effort-induced or stable angina are experienced in such conditions as physical activity, emotional excitement or any other cause of increased cardiac workload.

167.

A patient with fracture of his lower jaw was admitted to the maxillofacial department. It was decided to fix his bones surgically under anaesthetic. After intravenous introduction of muscle relaxant there arose short fibrillar contractions of the patient’s facial muscles. What muscle relaxant was applied?

Explanation

Dithylinum is a neuromuscular blocking drug (myorelaxants). It attaches to the N-cholinoreceptor depolarizing the junction and providing a constant stimulation of the receptor. The continued binding of the depolarizing agent renders the receptor incapable of transmitting further impulses. They are used to relax muscles during surgery.

168.

Autopsy of a 56 y.o. man revealed in the right temporal part of brain a big focus of softened grey matter that was semi-liquid and light grey. Arteries of cerebral tela contain multiple whitish-yellow thickenings of intima that abruptly narrow the lumen. What is your diagnosis?

Explanation

        Stroke is sudden loss of blood circulation to an area of the brain resulting in a corresponding loss of neurologic function. Types:

·        Ischemic (70-80% of cases):

- Atherosclerotic (thrombotic): most common type

- Embolic

·        Intracerebral hemorrhage

·        Subarachnoid hemorrhage

·        Lacunar stroke

Atherosclerotic (thrombotic) stroke: Ischemic type of stroke is commonly caused by platelet thrombosis that develops over a disrupted atherosclerotic plaque. Characteristics: it usually develops at the periphery of the cerebral cortex, swelling of brain, loss of dermacation between gray and white matter, breakdown of myelin, pale infarct; Cystic area develops after 10days to 3weeks due to liquefactive necrosis.

169. Vitamin A deficit results in the impairment of twilight vision. Name the cells that have the above-mentioned photoreceptor function:

Explanation

Retina contains the visual receptors – rods and cones are composed of 4 structures: outer segment, inner segment, cell body and synaptic terminal. The outer segment of rod cell (which is the terminal portion as described in the question) is formed by the modified cilia and it contains a pile of freely floating flat membranous disks. There are about 1000 disks in each rod. Disks in rod cells are closed structures and contain the photosensitive pigment, the rhodopsin. The outer segment of cones is small and conical. It does not contain separate membranous disks as in rods. In cone, the infoldings of cell membrane form saccules, which are the counterparts of rod disks.

Rhodopsin or visual purple is the photosensitive pigment of rod cells. It is made up of a protein called opsin and a chromophore. Opsin present in rhodopsin is known as scotopsin. Chromophore is a chemical substance that develops colour in the cell. Chromophore present in the rod cells is called retinal. Retinal is the aldehyde of vitamin A or retinol. Vitamin A is the name given to a group of related compounds that include retinol (vitamin A alcohol); retinal (vitamin A aldehyde) and retinoic acid (vitamin A acid). Rod cells are responsible for dim light vision or night vision or scotopic vision (twilight vision).

170.

Neurological examination of a 65 y.o. patient revealed a haemorrhage within the superior temporal gyrus. In the blood supply area of which artery is it?

Explanation

The middle cerebral artery is the greatest branch of the internal carotid artery. It runs along the lateral sulcus giving numerous branches. It supplies the lateral surface of the brain (parietal and frontal lobe) and the temporal pole.

171.

A 70 y.o. man has cut an abscess off in the area of mammiform process during shaving. Two days later he was admitted to the hospital with inflammation of arachnoid membranes. How did the infection penetrate into the cavity of skull?

Explanation

The emissary veins communicate the dural venous sinuses with the extrinsic veins of head. They pass within the respective cranial canals. The largest emissary veins are parietal, mastoid, condylar and occipital emissary veins. The mastoid emissary vein (vena emissaria mastoidea) passes within the mastoid foramen. It communicates the transverse sinus with the occipital vein.

172. A 22 y.o. woman has enlarged lymph nodes. Histologically: a lymph node contains lymphocytes, histiocytes, reticular cells, small and big Hodgkin’s cells, multinucleated Sternberg cells, isolated foci of caseous necrosis. What disease are these changes typical for?

Explanation

       Hodgkin’s disease or lymphogranulomatosis is a disorder involving primarily the lymphoid tissue. It is separated from non-hodgkin lymphoma because:

First, it is characterized morphologically by the presence of distinctive neoplastic giant cells called Reed-sternberg’s cells, admixed with a variable inflammatory infiltrate.

Secondly, it is often associated with distinctive clinical features, including systemic manifestations such as fever.

Reed-sternberg cell: classically, it is a large cell, most often binucleate or bilobed, with two halves often appearing as mirror images of each other. Its identification is essential for the histologic diagnosis. The origin of Hodgkin’s lymphoma is unknown.

173.

A lymph node punctate of a patient with suspected protozoal disease was examined. Examination of the stained specimen (Romanovsky’s stain) revealed some crescent bodies with pointed end, blue cytoplasm and red nucleus. What protozoan were revealed in the smears?

Explanation

ToRCHeS infections: Toxoplasma gondii; Rubella; Cytomegalovirus; HIV; Herpes simplex virus-2; Syphilis. These are microbes that may pass from mother to fetus. Transmission is transplacental in most cases or via delivery. Toxoplasma gondii causes toxoplasmosis. It is transmitted from cat faeces or ingestion of undercooked meat. Congenital infection of the fetus occurs only when the mother is infected during pregnancy. Congenital infection can result in abortion, stillbirth or neonatal disease with encephalitis, chorioretinitis and hepatosplenomegaly.

174. During pubescence the cells of male sexual glands begin to produce male sex hormone testosterone that calls forth secondary sexual characters. What cells of male sexual glands produce this hormone?

Explanation

Early in male development, mesenchyme separating the seminiferous cords gives rise to leydig (interstitial) cells that produce testosterone to stimulate development of the indifferent primordium into a testis. Leydig cells are large, polygonal, eosinophilic cells that typically contain lipid droplets. Leydig cells differentiate and secrete testosterone during early fetal life. Secretion of testosterone is required during embryonic development, sexual maturation and reproductive function:

·        In the embryo, secretion of testosterone and other androgens is essential for the normal development of the gonads in the male fetus.

·        At puberty, secretion of testosterone is responsible for the initiation of sperm production, accessory sex gland secretion and development of secondary sex characteristics.

·        In the adult, secretion of testosterone is essential for the maintenance of spermatogenesis and of secondary sex characteristics, genital excurrent ducts and accessory sex glands.

175.

An isolated cell of human heart automatically generates excitation impulses with frequency 60 times pro minute. What heart structure was this cell obtained from?

Explanation

The sinoatrial (SA) node is a small strip of modified cardiac muscle, situated in the superior part of lateral wall of right atrium, just below the opening of superior vena cava. The fibers of this node do not have contractile elements. SA node is called the pacemaker because the rate of production of impulse (rhythmicity) is higher in SA node than in other parts. The rate is 60-100/min and higher during tachycardia when stimulated by sympathetic effects. Atrioventricular (AV) node is 40-60/min.

176. Examination of a patient revealed a strong, balanced, inert type of higher nervous activity according to Pavlov. What temperament type does the patient have (according to Hippocrates classification)?

Explanation

Hippocrates classification due to temperament and social behavior: choleric, phlegmatic; sanguine; melancholic.

·        Choleric: egocentric and extroverts, excitable, impulsive and restless (dry and hot).

·        Phlegmatic: reasonable, calm, patient, caring and tolerant (moist and cold)

·        Sanguine: lively, sociable, carefree, talkative and pleasure seeking (hot and moist)

·        Melancholic: serious, introvert, cautious or even suspicious (cold and dry)

177.

A patient has a haemorrhage into the posterior central gyrus. What type of sensitivity on the opposite side will be disturbed?

Explanation

The postcentral gyrus (posterior central gyrus) is delimited anatomically by the central sulcus anteriorly and the postcentral sulcus posteriorly. It is the primary sensory area responsible for perception and integration of cutaneous (skin) and kinesthetic (proprioceptive) sensations. It receives sensory impulses from cutaneous receptors (touch, pressure, pain, temperature) and proprioceptors of opposite side through thalamic radiation.

Precentral gyrus – Primary motor area

178.

Analysis of a punction biopsy material of liver revealed hepatocyte dystrophy with necroses as well as sclerosis with disorder of beam and lobulous structure, with formation of pseudolobules and regenerative nodes. What is the most probable diagnosis:

Explanation

Cirrhosis of liver is the final stage of liver disease and is defined by 3 characteristics:

·        Fibrosis is present in the form of delicate bands or broad scars replacing multiple adjacent lobules.

·        The parenchymal architecture of the liver is divided by interconnecting fibrous scars (disorder of lobule structure).

·        Parenchymal nodules are created by regeneration of hepatocytes. The nodules may vary from micronodules (<3mm in diameter) to macronodules (>3mm in diameter).

Morphological pattern: the liver is small (dystrophy), having distorted shape with irregular and coarse scars and nodules. Active liver cell necrosis is observed. Liver cells vary considerably in size and multiple large nuclei are common in regenerative nodules. Fatty degeneration may be present.

179. Examination of a 60 y.o. patient revealed hyperglycemia and glucosuria. A doctor administered him a medication for internal use. What medication is it?

Explanation

Sulfonylurea are insulin secretagogues because they promote insulin release from the β-cells of the pancreas. The primary drugs used today are the 2nd generation drugs – glibenclamide, glipizide, glimepiride etc. These agents are useful in the treatment of patients who have type 2 diabetes mellitus and cannot be managed by diet alone. Metformin is the only currently available biguanide; its classed as an insulin sensitizer. Acarbose – α glucosidase inhibitor. Butamide (tolbutamide) is also a sulfonylurea but is older and glibenclamide is more potent and used much more commonly. Actrapid (soluble insulin) is fast acting but not a sulfonylurea.

180.

A patient who had been working hard under conditions of elevated temperature of the environment has now a changed quantity of blood plasma proteins. What phenomenon is the case?

Explanation

Liver is the primary organ of protein synthesis. In a case of dysfunction of protein synthesis, hypoproteinemia occurs. We have absolute and relative hypoproteinemia. Absolute is when there is disturbed synthesis (absolute hypoproteinemia) and in cases of increased synthesis (absolute hyperproteinemia); while relative does not have to do with synthesis but other pathologies e.g. renal lesions, in which case, there is loss of protein with urine (relative hypoproteinemia); also vomiting, diarrhea, profuse sweating can produce relative hyperproteinemia.

It is Relative because it does not have to do with synthesis. The man must have lost a lot of fluid through sweat, thereby decreasing the fluid content of the body and creating a relative hyperproteinemia. Decrease blood volume (liquid component alone) → ↑concentration of blood.

181.

An experimental rat with extremity paralysis has no tendon and cutaneous reflexes, muscle tone is decreased, but muscles of the affected extremity maintain their ability to react with excitation to the direct action of continuous current. What type of paralysis is it?

Explanation

Paralysis is the absence or decrease of muscular strength in limbs which makes movement impossible. It can be central or peripheral. Central paralysis caused by central motor neuron lesion, with symptoms such as spastic paralysis-muscular hypertonus (muscle tone), proprioceptive reflexes increase, absence of atropy. Peripheral paralysis caused by peripheral motor neuron lesions with symptoms such as atonic, flaccid paralysis-muscular atrophy and no reflexes.

182.

A liquidator of a breakdown at a nuclear power plant who was irradiated complained about vomiting that occurs all of a sudden. What medication should be prescribed?

Explanation

        Metoclopramide is an antiemetic drug; an antidopamine. It blocks D2-dopamine receptors of emetic center in the CNS. It has a central antiemetic effect in vomiting of any genesis. Uses – vomiting induced by irritation of gastrointestinal mucous membrane, post-operative period, pregnancy toxicosis, radiation sickness etc.

183. Urine examination of a patient with acute cystitis revealed leukocytes and a lot of gram-negative bacilli. Inoculation resulted in growth of colonies of mucous nature that formed green soluble pigment. What microorganism is the most probable cause of the disease?

Explanation

Pseudomonas produces blue green pigment; Yellow-green and red-brown pigments. Escherichia, proteas, Klebsiella, Salmonella do not produce pigments.

184. A 45 y.o. patient consulted a doctor about plaque-shaped formation on his neck. Histological examination of biopsy skin material revealed tumourous cells of round and oval form with thin ring of basophilic cytoplasma that resemble of cells of basal epidermal layer. What tumour is it?

Explanation

Skin cancer (epithelial tumors) are divided into 4 groups:

·        Intraepidermal cancer (cancer in situ): Bowen’s disease, erythroplasia of Queyrat.

·        Basal cell carcinoma (BCC, basalioma)

·        Epidermoid cancer

·        Skin appendage cell carcinoma: Paget’s disease

Basal cell carcinoma (basalioma) is malignant; the tumor is characterized by slow growth (sometimes for years), locally destructive invasive growth without metastases and frequent recurrences. The tumor is of polymorphous structure derived from the epidermis and skin appendages. The most prevalent location is the face, neck and scalp. Gradually increasing in size, they can form a large plaque or node with ulcerative depression in the center and elevated borders. It has the appearance of a flat well-defined reddish, round or oval-shaped spot or plaque.

185. A patient has pain, edema and reddening of his skin in the anterosuperior area of his thigh and his foot’s thumb. What lymph nodes of his lower extremity responded to the inflammatory process?

Explanation

The inguinal lymph nodes are divided into superficial and deep. The superficial lymph nodes reside outside the fascia lata below the inguinal ligament and around the saphenous opening. The superficial inguinal lymph nodes drain the lower limb, skin and fascia of the external genitalia, perineum, buttocks and abdominal wall. The efferent vessels pass to the deep inguinal nodes.

186.

A laboratory received a material from a patient’s wound. Preliminary diagnosis is gaseous gangrene. What microbiological method should be applied to determine species of causative agent?

Explanation

The laboratory diagnosis of infectious diseases involves two main approaches: one is the bacteriologic approach in which the organism is identified by staining and culturing the organism and the other is the immunologic approach in which the organism is identified by detection of antibodies against the organism in the patient’s serum. When cultures are negative (i.e bacteriologic method), then immunologic testing is commonly used.

187.

Examination of cell culture got from a patient with lysosomal pathology revealed accumulation of great quantity of lipids in the lysosomes. What of the following diseases is this disturbance typical for?

Explanation

Tay- sachs disease is a progressive neurodegeneration, developmental delay, “cherry-red” spot on macula, lysosomes with onion skin, no hepatosplenomegaly. It is an autosomal recessive disease, caused by the deficiency of hexosaminidase A resulting in the accumulation of GM2 ganglioside in organs of the body (e.g. brain, liver, spleen).

188.

A patient got a craniocerebral trauma that resulted in right-side convergent strabismus. Damage of which craniocerebral nerve caused such consequences?

Explanation

Lateral rectus muscle: Abducts the eyeball (pulls it away from the midline) & it’s supplied by the abducent nerve (CN VI) which abducts the eye, pulling it laterally. A lesion of CN VI will lead to failure of eye to abduct (strabismus).

189. In case of a penetrating wound of the anterior abdominal wall the wound tract went above the lesser curvature of stomach. What peritoneum formation is most likely to be injured?

Explanation

The lesser omentum is the residue of dorsal mesentery of stomach and duodenum. It consists of two main ligaments: hepatogastric and hepatoduodenal ligaments. Others are hepatophrenic , hepatoesophageal and hepatocolic ligaments.

·        Hepatogastric ligament (ligamentum hepatogastricum) is a double layer of peritoneum running from the porta hepatica to the lesser curvature.

·        Hepatoduodenal ligament (ligamentum hepatoduodenale) running from the porta hepatica to the superior part of the duodenum. It also consists of double layer of peritoneum and enfolds the hepatic portal vein, hepatic artery proper and common bile duct.

190.

A rabbit’s nerve that innervates the right ear was cut and its right superior cervical ganglion was removed. Immediately after operation the temperature of ear skin was measured. It was revealed that the temperature of the rabbit’s ear skin on the side of denervation was by 1, 50C higher than on the opposite intact side. What of the following is the most probable explanation of the above-mentioned effects?

Explanation

·        Neuroparalytic arterial hyperemia is caused by damage or blockage of α-adrenoreceptors (sympathetic nervous system). It is characterized by reduction or absence (paralysis) of the sympathetic nervous system effects on the walls of the arteries and arterioles.

·        Neurotonic arterial hyperemia is caused by activation of parasympathetic nervous system; irritation of vascular dilators part of vascular center (CNS) or inhibition of vascular – constrictor part of this center (vasomotor center of CNS); M-cholinoreceptors; H2-histaminereceptors. It is characterized by predominance of the parasympathetic nervous system effects on arterial vascular walls.

191. A 63 y.o. man fell ill with acute tracheitis and bronchitis accompanied by bronchial pneumonia. On the 10th day the patient died from cardiopulmonary insufficiency. Autopsy revealed fibrinous hemorrhagic laryngotracheobronchitis; lungs were enlarged, their incision revealed the \\\"coal-miner’s\\\" effect caused by interlacing of sections of bronchial pneumonia, hemorrhages into the pulmonary parenchyma, acute abscesses and atelectases. Internal organs have discirculatory and dystrophic changes. What is the most probable diagnosis?

Explanation

Influenza is an acute respiratory viral infection. The clinical-morphological forms of influenza are slight influenza, mild influenza and severe influenza. The histopathologic features of mild influenza include a necrotizing tracheitis and bronchitis; diffuse hemorrhagic necrotizing pneumonitis with pulmonary edema. Pulmonary complications in severe influenza: lungs are dark and firm with interstitial emphysema (enlarged lungs); necrotic foci; abscesses; lungs appear as “large variegated (motley) influenza lung.” Obliterating bronchitis, bronchiolitis, bronchiectasis and other chronic lung diseases can develop.

192.

Autopsy of a man who died from influenza revealed that his heart was slightly enlarged, pastous, myocardium was dull and had specks. Microscopical examination of myocardium revealed signs of parenchymatous adipose and hydropic dystrophy; stroma was edematic with poor macrophagal and lymphocytic infiltration, vessels were plethoric; perivascular analysis revealed petechial hemorrhages. What type of myocarditis was developed in this case?

Explanation

193.

A patient with chronic cardiac insufficiency has been treated with cardiotonic drugs and a thiazide diuretic, but in spite of it there are still edemata and risk of ascites. What medication should be prescribed to amplify diuretic effect of the applied drugs?

Explanation

K+ sparing diuretics:

  - aldosterone antagonist: spironolactone, eplerenone

  - inhibit Na+ reabsorption: triamterene

The answer is Spironolactone because after the administration of cardiotonics and a thiazide diuretic, if the edema still persists, it is probably due to an increased action of aldosterone on the principal cells in collecting tubule of the kidney. In this case, an aldosterone antagonist will enhance the diuretic effect to prevent the occurrence of ascites. Blocking aldosterone effect prevent further reabsorption of Na+ and H2O which will definitely enhance the diuretic effect. Aldosterone acts on mineralocorticoid receptor → mRNA → protein synthesis (synthesis of Na+ channels). Spironolactone prevents/blocks the synthesis of Na+ channels.

194. A patient ill with collagenesis has been taking prednisolone for a long time. Hypokaliemia development caused spastic pain of skeletal muscles. What medication should be used in order to correct potassium exchange?

Explanation

Panangin is an antidote; a potassium preparation. It is indicated in hypokalemia. It is used to replace potassium concentration in the body. It can also be used as an antiarrhythmic drug.

195.

A boy is 7 y.o. Objectively: against the background of hyperemic skin there is knobby bright-pink rash on his forehead, neck, at the bottom of abdomen, in the popliteal spaces; nasolabial triangle is pale. Examination of oropharyngeal surface revealed localized bright-red hyperemia; tonsils are swollen, soft, lacunas contain pus, tongue is crimson. Cervical lymph nodes are enlarged, dense and painful. What is the most probable diagnosis?

Explanation

Scarlet fever is an acute infectious disease accompanied by local inflammatory changes mainly in the pharynx and typical generalized rash.

* Local changes: primary scarlatnic affect/complex – catarrhal or necrotic tonsillitis. Catarrhal tonsillitis: hyperemia of pharynx (flaring pharynx or burning faucet). Necrotic tonsillitis: coagulative necrosis and ulceration.

* General changes are first of all RASH. The face is also involved, but usually a small area about the mouth (nasolabial triangle) remains relatively unaffected to produce a circumoral pallor. Tongue is beefy red/crimson red and glistening, strawberry tongue.

196.

A patient complains of dryness of head skin, itching, fragility and loss of hair. After examination he was diagnosed with seborrhea. Disturbed activity of which cells caused this condition?

Explanation

Seborrhea is a chronic relapsing but usually mild dermatitis. It is an inflammatory skin disorder affecting the scalp, face and torso. Typically, it presents with scaly, flaky, itchy and red skin. It particularly affects the sebaceous-gland-rich areas of skin. It is a malfunction of the sebaceous glands; And vitamin imbalance play a leading role in the pathogenesis of this disease.

197.

In the surgical department of a hospital there was an outbreak of hospital infection that showed itself in often postoperative wound abscesses. Bacteriological examination of pus revealed aurococcus. What examination shall be conducted to find out the source of this causative agent among the department personnel?

Explanation

198. Short-term physical activity resulted in reflex amplification of heart rate and raise of systemic arterial pressure. What receptors activation was the main cause of pressor reflex realization?

Explanation

Proprioreceptors are the receptors which give response to change in the position of the body. These receptors are situated in joints, tendons and muscles. Proprioreceptors are stimulated during the muscular exercise and send impulses to the brain, particularly cerebral cortex, through somatic afferent nerves. Cerebral cortex in turn causes increase in heart rate, force of contraction of heart, arterial pressure, hyperventilation by sending impulses to medullary vasomotor and respiratory centers.

199.

In course of an experiment a skeletal muscle is being stimulated by a series of electric impulses. What type of muscle contraction will arise, if every subsequent impulse comes in the period of relaxation of single muscle contraction?

Explanation

Tetanus is defined as the sustained contraction of muscle due to repeated stimuli with high frequency. When the multiple stimuli are applied at a higher frequency in such a way that the successive stimuli fall during contraction period (period of shortening) of previous twitch, the muscle remains in state of tetanus. While increasing the frequency, fusion of contractions increases every time and finally complete tetanus occurs. Holotetanus – complete tetanus.

When the frequency of stimuli is not sufficient to cause tetanus, the fusion of contractions is not complete and then we have – incomplete/partial/waved tetanus. This occurs (i.e. the next stimuli occurs) during the period of relaxation.

200. A patient who suffers from acute myocarditis has clinical signs of cardiogenic shock. What of the under-mentioned pathogenetic mechanisms play the main part in shock development?

Explanation

Cardiogenic shock is the type of shock caused by the failure of the heart to pump effectively. This can be due to damage to the heart muscle, most often from a large myocardial infarction. Other causes of cardiogenic shock include arrhythmias, cardiomyopathy, myocarditis, congestive heart failure or cardiac valve problems.